Вы находитесь на странице: 1из 94

INS INSIGHTS IAS PRELIMS TEST SERIES - 2017

Prelims 2017 – Test 28

SOLUTIONS
1. Consider the following about T-Phi Grams.
1. They are an indicator of parasite density in disease prone
regions.
2. They can be derived From INSAT satellite systems.
3. They can be used to understand atmospheric profiles.
Select the correct answer using the codes below.

a) 1 only
b) 2 and 3 only
c) 2 only
d) 1, 2 and 3
Solution: b)
Justification: Statement 1: A tephigram is one of the thermodynamic
diagrams commonly used in weather analysis and forecasting.

The name evolved from the original name to describe the axes of
temperature (T) and entropy used to create the plot.

Statement 2: IMD derives T-Phi Grams using the INSAT-3D Sounder. It


is useful for predicting cyclones.

Some important quantities found on an atmospheric profile plotted on a


tephigram: such as dewpoint temperature; level of free convection;
amount of water vapour condensed in a parcel lift etc.

Statement 3: Usually, temperature and dew point data from radiosondes


are plotted on these diagrams to allow calculations of convective stability
of air parcels indicating the winds at different heights.

Q Source: http://satellite.imd.gov.in/map_skm2.html

2. Consider the following statements.


1. Capital punishment or Death Sentence is impermissible under
the Universal Declaration of Human Rights (UDHR).

WWW.INSIGHTSIAS.COM 1
INS INSIGHTS IAS PRELIMS TEST SERIES - 2017

2. Revocation of appointment on grounds of pregnancy amounts


to violation of women’s rights under Convention on the
Elimination of all forms of Discrimination against Woman
(CEDAW).
Which of the above is/are correct?

a) 1 only
b) 2 only
c) Both 1 and 2
d) None
Solution: c)
Justification: Statement 1: The United Nations adopted without
dissent the Universal Declaration of Human Rights (UDHR). The
Declaration proclaims the right of every individual to protection from
deprivation of life. It states that no one shall be subjected to cruel or
degrading punishment. The death penalty violates both of these
fundamental rights.

Statement 2: It provides that, “States Parties shall ensure to women


appropriate services in connection with pregnancy, confinement and the
post-natal period, granting free services where necessary, as well as
adequate nutrition during pregnancy and lactation”

It also provides for special protection for women from harmful types of
work during pregnancy and with the provision of paid maternity leave.

Q Source: Rights Issue: International Conventions

3. As an individual, the maximum rate of income tax that someone


has to effectively pay is
a) Not more than 20%
b) More than 20%, but not more than 30%
c) More than 30%, but not more than 40%
d) More than 40%
Solution: b)

WWW.INSIGHTSIAS.COM 2
INS INSIGHTS IAS PRELIMS TEST SERIES - 2017

Justification: The highest income tax rates are 30% above Rs. 10
Lakhs of annual income. But, this slab rate is applicable only for income
that exceeds ten lakhs, even if total income is more than ten lakhs.

Suppose you earn 30 lakhs. You don’t pay 30% of 30 lakhs, i.e. 9 lakhs in
taxes. You pay 5% of 2,50,000 (income between 2.5 and 5 lakhs), 20% of
5 lakhs (income between 5 and 10 lakhs), and then 30% of 20 lakhs
(income above 10 lakhs in this case). So, maximum tax liability can be Rs
1,250+1,00,000+6,00,000 which is just more than 7 lakhs.

So, the total figure is between 20% to 30%.

Q Source: Annual Budget 2017-18: Income tax slab Revision

4. The Sapru committee report that was published in 1945 advocated


that
1. Fundamental Rights must be included in the Constitution of
India.
2. Princely states should be integrated in the Indian Union without
giving them privy purse.
Which of the above is/are correct?

a) 1 only
b) 2 only
c) Both 1 and 2
d) None
Solution: a)
Justification: Statement 1: This committee divided fundamental rights
into two parts viz. Justifiable Rights and Non-justifiable rights. The
Justifiable rights were those enforceable by a court of law.

These enforceable rights were incorporated in the Part III of the


Constitution.

WWW.INSIGHTSIAS.COM 3
INS INSIGHTS IAS PRELIMS TEST SERIES - 2017

The non-justifiable rights were incorporated as a directive to the state to


take all measures to provide those rights to individuals without any
guarantee.

Statement 2: There was no such provision, in fact even the word privy
purse wasn’t mentioned in the report.

Q Source: Improvisation: Past year UPSC papers

5. As per the Notification on lead content in Household and


Decorative paints Rules, 2016 by the Ministry of Environment,
Forest and Climate Change (MoEFCC)
1. Imports of lead containing paint shall be banned in the country
effective January 1, 2017.
2. Domestic paints can contain maximum 9000 parts per million
of lead of the weight of the total non-volatile paint content.
Which of the above is/are correct?

a) 1 only
b) 2 only
c) Both 1 and 2
d) None
Solution: d)
Justification: Statement 1: Imports are not banned. But, the
notification says that paints containing lead:

• Their label shall contain the name and address of the manufacturer
or importer, as the case may be;
• Their label shall contain the date of manufacture or import into the
country, as the case may be.
Statement 2: 9000 parts per million (PPM) is a huge number and will be
highly toxic. You can eliminate statement 2.

WWW.INSIGHTSIAS.COM 4
INS INSIGHTS IAS PRELIMS TEST SERIES - 2017

Notification says that, “Household and Decorative Paints (hereinafter


referred to as product) containing lead or lead compounds (calculated as
lead metal) in excess of 90 parts per million (0.009 per cent.) of the
weight of the total non-volatile content of the weight of the dried paints
film is hereby prohibited.”

Q Source:
http://envfor.nic.in/sites/default/files/final%20notification_Lead%20i
n%20Paints.pdf

6. Consider the following colour settings presently being used by the


Survey of India for classification of land uses in India:
1. Yellow: A. Hills

2. Red: B. Settlements

3. Brown C. Cultivated Area

4. Black D. Road Transportation network

What is the correct match for the above?

a) 1C, 2B, 3A, 4D


b) 1A, 2C, 3D, 4B
c) 1B, 2A, 3D, 4A
d) 1A, 2B, 3C, 4D
Solution: a)
Justification: Black: all writings on the map except grid numbers
(names, abbreviations such as DB, RS, P0), river banks, broken ground,
dry streams, surveyed trees, heights and their numbering, railway lines,
telephone and telegraph lines, lines of latitude and longitude.

Blue: water features or water bodies (lakes, rivers, wells, ponds, ctc)

Green: wooded and forested areas shown with green wash; orchards,
scattered trees and scrubs shown in green colour.
WWW.INSIGHTSIAS.COM 5
INS INSIGHTS IAS PRELIMS TEST SERIES - 2017

Note: Surveyed trees are shown in black which serve as landmarks for
further survey related work and are not allowed to be cut.

Yellow: cultivated areas shown with a yellow wash

White patches: uncultivated land

Brown: contour lines, their numbering, form lines and sand features
such as hills and dunes

Red: grid lines and their numbering; wads, cart and foot tracks,
settlements, huts and buildings.

Q Source: Longman ICSE 10th Standard Geography: Chapter 1

http://tcpomud.gov.in/Divisions/MUTP/Land/Landuse_Classification_
Report.pdf

7. What is/are the benefits of issuing Soil Health card (SHC) to


Indian farmers?
1. It will help reduce the consumption of chemical fertilizers and
cost of crop production.
2. It will help reduce our trade deficit as well as fiscal deficit.
3. It will help tackle imbalanced fertilizer use which is causing
deficiency of nutrients in soil.
Select the correct answer using the codes below.

a) 1 and 3 only
b) 2 only
c) 1 only
d) 1, 2 and 3
Solution: d)
Concept: It is a field-specific printed report card issued to farmers in
once in three years indicating the status of his soil in terms of 12
parameters.

WWW.INSIGHTSIAS.COM 6
INS INSIGHTS IAS PRELIMS TEST SERIES - 2017

It is also accompanied by an advice on the various fertilizers and other


soil amendments he is suppose to make including in Major Nutrients,
pH (Soil Reaction; acidity / alkalinity of the soil), Total Dissolved salts
etc.

Justification: Statement 1: SHC will ensure that farmers do not spend


money unnecessarily on purchase of fertilizers by adding more than
required. Once there is economy on the use of chemical fertilizers, the
cost of production is expected to decrease.

Statement 2: India imports large quantity of various fertilizers to meet


the demand.

• About 25-30% of the requirement of Urea, 90% of the requirement


of Diammonium phosphate (DAP) and 100% of the requirement of
Muriate of Potash (MOP) are met by imports.
• The soil test based fertilizer usage will reduce import bill and will
also ensure higher yields per unit.
• Fertilizers also account for huge subsidy bills lowering of which
will reduce our fiscal deficit.
Statement 3: The existing Nitrogen - Phosphorous - Potassium (NPK)
consumption ratio in the country is skewed at 8.2:3.2:1 (2012-13) as
against the preferred ratio of 4:2:1.

• A great variability is observed in fertiliser consumption among


States from 250 kg /ha in Punjab to 2 kg / ha in Arunachal
Pradesh, during 2012-13.
• However, imbalanced application of fertilisers have caused
deficiency of primary nutrients (i.e. NPK), secondary nutrients
(such as sulphur), and micronutrients (boron, zinc, copper etc.), in
most parts of country.
Q Source: http://soilhealth.dac.gov.in/

8. A reputed US based international media house in the latest issue of


its publication ‘Condé Nast Traveller’ has listed the luxury river
cruise on X as one of the ‘six river cruises to take in 2017’. X is?
a) Cauvery
WWW.INSIGHTSIAS.COM 7
INS INSIGHTS IAS PRELIMS TEST SERIES - 2017

b) Ganga
c) Godavari
d) Indus
Solution: b)
Learning: The media house has placed the luxury cruise vessel Ganges
Voyager II which sails on the Ganga from Kolkata to Varanasi, in the
league of cruises on Mekong and Yangtze in China, Amazon in South
America, Volga in Russia and Irrawaddy in Myanmar.

Conde Nast’s endorsement of Ganga as a cruise destination is a shot in


the arm for river tourism in the country.

The Inland Waterways Authority of India facilitates cruise operations on


NW-1 (river Ganga) from Kolkata to Varanasi in collaboration with
private cruise operators.

This is the website of the travel magazine https://www.cntraveller.in/


(pictures of Ganga voyager II below)

WWW.INSIGHTSIAS.COM 8
INS INSIGHTS IAS PRELIMS TEST SERIES - 2017

Q Source: PIB Releases: 6th May 2017

9. Consider the following about South Asian demographics and


economy.
1. The most populous country also has the highest density of
population.
2. The country having the highest GDP (PPP) per capita is also the
most populous country.
Which of the above is/are correct?

a) 1 only
b) 2 only
c) Both 1 and 2
d) None
Solution: d)

WWW.INSIGHTSIAS.COM 9
INS INSIGHTS IAS PRELIMS TEST SERIES - 2017

Justification: Statement 1 Population density (people per sq. km) in


South Asia was last measured at around 365 in 2015. The order is
Bangladesh> India> SL and rest.

https://en.wikipedia.org/wiki/List_of_countries_and_territories_by_p
opulation_density

Statement 2: India has the highest GDP and GDP (PPP) and the highest
population in South Asia. But, Sri Lanka tops in terms of GDP (PPP) per
capita.

You can refer this list


https://en.wikipedia.org/wiki/List_of_Asian_countries_by_GDP_per_
capita

Q Source: South Asian Demographics

10. Why Potassium Permanganate is used for purifying drinking


water under many circumstances?
1. It is the most potent chemical disinfectant that is stable at room
temperature.
2. It is one of the strongest known reducing agents.
Which of the above is/are correct?

a) 1 only
b) 2 only
c) Both 1 and 2
d) None
Solution: d)
Justification: Statement 1 and 2: Potassium permanganate is an
oxidant, but a poor disinfectant.

1
WWW.INSIGHTSIAS.COM
0
INS INSIGHTS IAS PRELIMS TEST SERIES - 2017

Potassium permanganate is a basic treatment method that oxidizes


dissolved iron, manganese, and hydrogen sulfide into solid particles that
are filtered out of the water.

While it can also be used to control iron bacteria growth in wells, but as
mentioned earlier, it is not potent enough. So, 1 is wrong.

It is used in well water to control odor, colour and foul taste.

Q Source: Science 11th NCERT: Environmental Chemistry

11. Consider the following about Global Biodiversity Information


Facility (GBIF).
1. It is an international open data infrastructure funded by
governments providing a single point of access to large number
of records.
2. It encourages environmental institutions to publish data
according to common standards.
3. It funds all ecological research activities in major international
NGOs such as IUCN and Greenpeace.
Select the correct answer using the codes below.

a) 1 and 2 only
b) 2 only
c) 2 and 3 only
d) 1 and 3 only
Solution: a)
Justification: GBIF arose from a recommendation in 1999 by the
Biodiversity Informatics Subgroup of the Megascience Forum, set up by
the Organization for Economic Cooperation and Development (OECD).

Statement 1: It allows anyone, anywhere to access data about all types of


life on Earth, shared across national boundaries via the Internet.

It provides a single point of access (through this portal and its web
services) to hundreds of millions of records, shared freely by hundreds of

1
WWW.INSIGHTSIAS.COM
1
INS INSIGHTS IAS PRELIMS TEST SERIES - 2017

institutions worldwide, making it the biggest biodiversity database on


the Internet.

Statement 2: GBIF enables research not possible before, and informs


better decisions to conserve and sustainably use the biological resources
of the planet.

Statement 3: It does not fund these NGOs (in statement 3); Rather GBIF
operates through a network of nodes sharing and coordinating the
biodiversity information facilities of Participant countries and
organizations.

Q Source: https://www.gbif.org

12. There have been many concerns regarding Article 310 and Article
311 and even recommendations for their deletion from the
Constitution. These constitutional articles deal with
a) Civil Services
b) Ordinance making power of executive
c) Legislative Council in States
d) Special privileges extended to certain states
Solution: a)
Articles: A310 essentially conveys that civil servants of the Union hold
office during the pleasure of the President, and those at the state at the
pleasure of the Governor.

A311 conveys that they cannot be removed by an authority subordinate


to the one that appointed them, and that due inquiry must be conducted
where they are allowed to hear charges against them and defend
themselves appropriately.

Justification: It is believed by many, including the 2nd ARC that these


articles provide undue protection to dishonest and corrupt civil servants
1
WWW.INSIGHTSIAS.COM
2
INS INSIGHTS IAS PRELIMS TEST SERIES - 2017

and shield them from action. This is because the protection is excessive,
and hearings go through many bureaucratic layers, which ultimately
corrodes the fabric of administration and its efficiency.

There are many arguments against this as well, but delving into there is
not a subject of concern here.

Q Source: 2nd ARC Reports

13. Consider the following statements.


1. The Purusha-sukta of Rig Veda Samhita lays the order for a
classless society that lives in peace and harmony.
2. In the Panchavimsha Brahmana, Indra is associated with the
creation of Varnas.
3. The Chhandogya Upanishad mention that non-pious
Brahmanas be offered as victims in purushmedha sacrifices.
Select the correct answer using the codes below.

a) 1 only
b) 2 only
c) 1 and 3 only
d) 1, 2 and 3
Solution: b)
Justification: Statement 1: It mentions division of society in four class
groups – Brahama, Kshatriya, Vaishya and Shudra.

Statement 2: There seems to have been some ambiguity about the


relative positions of the higher varnas. In the Panchavimsha Brahmana
where Indra is associated with the creation of the varnas, the Rajanya are
placed first, followed by the Brahmana and Vaishya.

Others place different orders.

1
WWW.INSIGHTSIAS.COM
3
INS INSIGHTS IAS PRELIMS TEST SERIES - 2017

Statement 3: Although there are no clear indications of the practice of


untouchability in later Vedic texts, groups such as the Chandalas were
clearly looked on with contempt by the elites.

• The Chhandogya Upanishad and Taittiriya and Shatapatha


Upanishad mention the Chandala in a list of victims to be offered
in the presumably symbolic purusha-medha (human sacrifice), and
describe him as dedicated to the deity Vayu (wind).
• The dedication to Vayu has been interpreted as indicating that the
Chandala lived in the open air or near a cemetery, but this is far
from certain.
Q Source: Upinder Singh: A History of Ancient and Early Medieval
India: From the Stone Age to the 12th Century

14.Which among the following is NOT a widely accepted taxation


principle?
a) Canon of Equality
b) Canon of Economy
c) Canon of Uncertainty
d) Canon of Convenience
Solution: c)
Justification: Option C: A successful function of an economy requires
that the people, especially business class, must be certain about the sum
of tax that they have to pay on their income from work or investment.

The tax system should be such that sum of tax should not be arbitrarily
fixed by the income tax authorities.

Option A: Under the tax system based on equality principle the richer
persons in the society will pay more than the poor.

Option B: The Government has to spend money on collecting taxes levied


by it- Since collection costs of taxes add nothing to the national product,
they should be minimized as far as possible. If the collection costs of a
1
WWW.INSIGHTSIAS.COM
4
INS INSIGHTS IAS PRELIMS TEST SERIES - 2017

tax are more than the total revenue yielded by it, it is not worthwhile to
levy it.

Option D: Tax should not only be certain but the time and manner of its
payment should also be convenient to the contributor. For e.g. if land
revenue is collected at the time of harvest, it will be convenient since at
this time farmers reap their crop and obtain income.

Q Source: Improvisation: 12th NCERT Macroeconomics

15. Tropic of Cancer is closest to which of these cities in India?


a) Shimla
b) Ghaziabad
c) Nagpur
d) Gandhinagar
Solution: d)
Learning: Some cities that are closest to tropic of cancer are:

• Gandhinagar (Gujarat) - 23.10 N


• Bhopal (M.P) - 23.16 N
• Ranchi (Jharkhand) - 23.11 N
• Kolkata (W.B) - 22.34 N
• Agartala (Tripura) - 23.51 N
• Aizwal (Mizoram) - 23.36 N
Shimla and Ghaziabad are further up north, and Nagpur is quite south of
tropic of cancer. So, most appropriate option is D.

Q Source: Map based questions

1
WWW.INSIGHTSIAS.COM
5
INS INSIGHTS IAS PRELIMS TEST SERIES - 2017

16.Consider the following about Background Air Pollution Monitoring


Network (BAPMoN).
1. India is not a part of this network.
2. It is a part of Global Atmospheric Watch (GAW) Programme.
3. It is being conducted by World Meteorological Organization
(WMO).
Select the correct answer using the codes below.

a) 2 and 3 only
b) 3 only
c) 1 only
d) 1 and 2 only
Solution: a)
Justification: With a view to document the long term changes in
composition of trace species of the atmosphere as a result of changing
land use pattern, WMO had commissioned a global programme called
Background Air Pollution Monitoring Network (BAPM0N) which is now
a part of the Global Atmospheric Watch (GAW) Programme. So, 2 and 3
are correct.

• India had set up 10 such BAPMon stations. So, 1 is wrong.


• At these stations, rain water samples are collected every month and
these are sent to the Central Chemical Laboratory at Pune for
complete chemical analysis.
• Atmospheric turbidity which indicates the columnar aerosol load
of the atmosphere, is also measured at these stations using
sunphotmeters.
Q Source:
http://imdpune.gov.in/Clim_Pred_LRF_New/air/airpolln.html

17. In 2017, India will be hosting which of these international sports


events?
a) ICC Women’s Cricket World Cup
1
WWW.INSIGHTSIAS.COM
6
INS INSIGHTS IAS PRELIMS TEST SERIES - 2017

b) FIFA Under-17 World Cup


c) Davis Cup World Group Play-offs
d) Superbike World Championship
Solution: b)
Learning: Option A: 2016 ICC Women's World Twenty20 was held in
India.

Option B: FIFA Under-17 World Cup 2017 will be played in six cities,
Delhi, Navi Mumbai, Kolkata, Goa, Kochi and Guwahati, from October
6-28, 2017.

Option C: Davis Cup World Group Play-offs were organized in 2014 in


KSLTA Tennis Stadium, Bangalore.

Option D: It was to be organized in Buddh International Circuit, Noida


but was later cancelled.

Q Source: PIB Releases: 02nd May 2017

18. Which of the following statements is/are correct?


1. District Collector usually supervises the revenue department
and land acquisition in the district.
2. District Commissioner of Police usually grants licences for arms
and explosives in the district.
Which of the above is/are correct?

a) 1 only
b) 2 only
c) Both 1 and 2
d) None
Solution: c)

1
WWW.INSIGHTSIAS.COM
7
INS INSIGHTS IAS PRELIMS TEST SERIES - 2017

Justification: Statement 1: District Collectors are entrusted with a


wide range of duties in the jurisdiction of the district. While the actual
extent of the responsibilities varies in each State, they generally involve

As Collector: land assessment; land acquisition; collection of land


revenue; collection of income tax dues, excise duties, irrigation dues etc;
distribution of agricultural loans; Chairman of the District Bankers
Coordination Committee etc.

As District Magistrate: maintenance of law and order; supervision of the


police and jails etc.

Statement 2: The application for the grant of Arm License is submitted


in the office of DCP by the applicant or through his representative on the
prescribed Performa.

Where the licensing authority deems it necessary for the security of the
public, peace or for public safety it may refuse to grant such license.

Q Source: SSC Past year papers

19.During the reorganization of Indian states, it was separated from


Madras and organized into a separate union territory for
administrative purposes. It is also known as the coral group of
islands of India:
a) Minicoy
b) Lakshadweep
c) Andaman
d) Nicobar
Solution: b)
Learning: There are no conclusive theories about the formation of
these coral atolls. The most accepted theory is given by the English
Evolutionist Sir Charles Darwin.

1
WWW.INSIGHTSIAS.COM
8
INS INSIGHTS IAS PRELIMS TEST SERIES - 2017

He concluded in 1842 that the subsidence of a volcanic island resulted in


the formation of a fringing reef and the continual subsidence allowed
this to grow upwards.

When the volcanic island became completely submerged the atoll was
formed encircling the lagoon where, with the action of the wind, waves,
reef to currents and temperature, the coral islands were formed.

Q Source: 6th NCERT Geography

20. Consider the following statements.


1. Grahak Suvidha Kendra (GVK) help redress consumer
complaint against a trader based in another State.
2. Prime Minister chairs the Central Consumer Protection Council.
3. A statutory Consumer Forum Network has been established in
India at district, state and Central levels.
Select the correct answer using the codes below.

a) 1 only
b) 1 and 3 only
c) 2 and 3 only
d) 1 and 2 only
Solution: b)
Justification: Statement 1: Minister of Consumer Affairs, Food and
Public Distribution is the Chairman, and Minister of State, Consumer
Affairs, Food and Public Distribution is the Vice-Chairman.

Corresponding ministers at the State level are also its members.

Statement 2: It will help redress consumer complaint against a trader


based in another State (for instance, if one is claiming a refund which the
trader has refused and which the consumer believes he/she is entitled
to).
1
WWW.INSIGHTSIAS.COM
9
INS INSIGHTS IAS PRELIMS TEST SERIES - 2017

It can ask counterpart centre in the State of the trader/ service provider
to contact the trader on consumer’s behalf and try to find a solution to
his/her complaint.

Statement 3: Under the provisions of the CP Act, 1986 there is a three


tier quasi-judicial mechanism at the District, State and National levels to
provide simple and speedy resolution to consumer disputes. You can find
a description about the three levels readily on the internet or at the Q
Source.

Q Source:
http://consumeraffairs.nic.in/WriteReadData/userfiles/file/GSKs.pdf

http://consumeraffairs.nic.in/WriteReadData/userfiles/file/CCPC704.p
df

http://consumeraffairs.nic.in/forms/contentpage.aspx?lid=584

21. The Philosophy of ‘New Public Administration’ vouches


a) Privatization of all government activities
b) Laissez fairez state
c) Inculcation of values and equity in administration
d) Outsourcing of major governmental activities
Solution: c)
Justification & Learning: New Public Administration theory deals
with the following issues:

• Democratic citizenship; Refers directly to the belief in creating a


government where the "common man" have a voice in politics.
• Public interest; Refers to the collective common good within
society, to which is the main objective of public interest.

2
WWW.INSIGHTSIAS.COM
0
INS INSIGHTS IAS PRELIMS TEST SERIES - 2017

• Public policy; The means by which new public policy is enacted,


and introduced. Not limited in participation of the public but
encouraged involvement.
• Services to citizens; Providing and upholding the moral and ethical
standard in regards to meeting the needs of citizens through
institutions and bureaucracies.
When this movement swept public administration, people started seeing
administration as a change maker rather than merely a bureaucratic tool.

Q Source: Basics of Public Administration

22. Consider the following with reference to the latest guidelines


of the Reserve Bank of India (RBI) on NBFCs.
1. NBFC should be registered under the Companies Act.
2. Any company, other than a bank, that deals with financial
products shall be deemed as a NBFC.
3. NBFCs can neither accept demand deposits nor make
investments.
4. NBFCs do not form part of the payment and settlement system
and thus cannot issue cheques drawn on itself.
Select the correct answer using the codes below.

a) 1 and 4 only
b) 1, 2 and 3 only
c) 2, 3 and 4 only
d) 1 and 2 only
Solution: a)
Definition: A Non-Banking Financial Company (NBFC) is a company
registered under the Companies Act, 1956 engaged in the business of
loans and advances, acquisition of
shares/stocks/bonds/debentures/securities issued by Government or
local authority or other marketable securities. So, 1 is correct.

• These can be of a like nature, leasing, hire-purchase, insurance


business, chit business but does not include any institution whose
principal business is that of agriculture activity, industrial activity,

2
WWW.INSIGHTSIAS.COM
1
INS INSIGHTS IAS PRELIMS TEST SERIES - 2017

purchase or sale of any goods (other than securities) or providing


any services and sale/purchase/construction of immovable
property.
Justification: Statement 2: Financial activity as principal business is
when a company’s financial assets constitute more than 50 per cent of
the total assets and income from financial assets constitute more than 50
per cent of the gross income. A company which fulfils both these criteria
will be registered as NBFC by RBI.

Statement 3 and 4: NBFCs lend and make investments and hence their
activities are akin to that of banks (so, 2 is wrong); however there are a
few differences as given below:

• NBFC cannot accept demand deposits; and moreover, as they do


not form part of the payment and settlement system, they cannot
issue cheques drawn on itself.
• Deposit insurance facility of Deposit Insurance and Credit
Guarantee Corporation is not available to depositors of NBFCs,
unlike in case of banks.
Q Source: https://www.rbi.org.in/Scripts/FAQView.aspx?Id=92

23. Consider the following statements.


1. Rashtriya Mahila Kosh (RMK) provides loans to NGO-MFIs
which on-lend to women Self Help Groups (SHGs).
2. National Commission for Women (NCW) has all the powers of a
civil court while hearing cases of harassment of women and
ordering appropriate punishments.

2
WWW.INSIGHTSIAS.COM
2
INS INSIGHTS IAS PRELIMS TEST SERIES - 2017

Which of the above is/are correct?

a) 1 only
b) 2 only
c) Both 1 and 2
d) None
Solution: a)
Justification: Statement 1: Established in 1993 is a national level
organization as an autonomous body under the aegis of the Ministry of
Women and Child Development, the operating model currently followed
by RMK is that of a facilitating agency.

RMK extends micro-credit to the women in the informal sector through


a client friendly, without collateral and in a hassle-free manner for
income generation activities.

Statement 2: It is an advisory body, not a court. It can attend to


grienvances but only recommend action, and not order punishments.

It can review the Constitutional and Legal safeguards for women;


recommend remedial legislative measures; facilitate redressal of
grievances and advise the Government on all policy matters affecting
women.

Q Source: http://wcd.nic.in/

24. Consider the following statements about Festivals of India


(FoIs) conducted by the government of India abroad.
1. Ministry of External Affairs is responsible for conducting
Festivals of India (FoIs) abroad.
2. It is conducted in South Asian countries to extend friendship
and build cultural cooperation.

2
WWW.INSIGHTSIAS.COM
3
INS INSIGHTS IAS PRELIMS TEST SERIES - 2017

3. It was recently nominated by the Government of India for


enshrining in the UNESCO Intangible heritage list.
Select the correct answer using the codes below.

a) 2 only
b) 1 and 3 only
c) 1 only
d) None of the above
Solution: d)
Justification: Statement 1: Ministry of culture is responsible for
conducting Festivals of India (FoIs) abroad as part of focused strategy
for strengthening international cultural relations and enhancing India's
soft power.

The events in Festival of India generally include dance, music, theater,


films, food, literary festival, yoga demonstration, folk arts, performing
arts, exhibitions and workshops etc.

Statement 2: Festival of India is being organized in different countries


such as Confluence: Australia; Urafiki Utsav: Kenya; Ukraine; Senegal;
Netherlands etc.

However, no South Asian country has been chosen so far.

Statement 3: There is no such news.

Q Source: http://www.indiaculture.nic.in/festivals-india-abroad

http://timesofindia.indiatimes.com/good-governance/centre/Festival-
of-India-Ministry-of-culture-invites-proposal/articleshow/53113478.cms

25. Consider the following about Liquified Petroleum Gas (LPG).

1. It is composed largely of methane and ethylene.


2. It can evaporate under room temperature and pressure.
2
WWW.INSIGHTSIAS.COM
4
INS INSIGHTS IAS PRELIMS TEST SERIES - 2017

3. It is lighter than air.


Select the correct answer using the codes below.

a) 2 only
b) 1 and 3 only
c) 3 only
d) 1, 2 and 3
Solution: a)
Justification: Statement 1: It consists mainly of propane, propylene,
butane, and butylene in various mixtures. It is produced as a by-product
of natural gas processing and petroleum refining.

Statement 2: At normal temperatures and pressures, LPG will


evaporate. Because of this, LPG is stored in pressurised steel bottles.

Statement 3: Unlike natural gas, LPG is heavier than air, and thus will
flow along floors and tend to settle in low spots, such as basements.

If used as a vehicle fuel, one challenge with LPG is that it can vary widely
in composition, leading to variable engine performance and cold starting
performance.

Q Source: Improvisation: 10th Science NCERT

26. While travelling through Nilgiri Hills, you are least likely to
encounter which of these tree species?
a) Sal
b) Pine
c) Eucalyptus
d) Cypress
Solution: a)
Justification: Option A: Sal tree is a deciduous tree. Nilgiris are
evergreen forests.

Option B: It generally occurs at an altitude of 1,600–6,600 ft in Nilgiri


forests.

2
WWW.INSIGHTSIAS.COM
5
INS INSIGHTS IAS PRELIMS TEST SERIES - 2017

Option C: Eucalyptus plantations have been raised in Nilgiri and


Kodaikanal hills in the past to meet firewood needs.

Option D: You can find these trees planted along highways in Nilgiris.

Q Source: SSC Past year Papers

27. Consider the following statements about Vijayanagar empire.


1. The rulers depended on import of horses from Arabia and
Central Asia as horses could not be domestically procured from
any reliable source.
2. The military commanders appointed by rulers were given
territories to govern as well as rights to collect taxes and dues
from peasants in their area.
Which of the above is/are correct?

a) 1 only
b) 2 only
c) Both 1 and 2
d) None
Solution: b)
Justification: Statement 1: As warfare during these times depended
upon effective cavalry, the import of horses from Arabia and Central Asia
was very important. Local communities of merchants known as kudirai
chettis or horse merchants provided horses to Vijayanagara rulers. So, 1
is wrong.

Statement 2: The amara-nayakas were military commanders who were


given territories to govern by the rayas.

• They collected taxes and other dues from peasants, craftspersons


and traders in the area.
• They retained part of the revenue for personal use and for
maintaining a stipulated contingent of horses and elephants.
• The amara-nayakas sent tribute to the king annually and
personally appeared in the royal court with gifts to express their
loyalty.

2
WWW.INSIGHTSIAS.COM
6
INS INSIGHTS IAS PRELIMS TEST SERIES - 2017

• Kings occasionally asserted their control over them by transferring


themfrom one place to another.
Q Source: 12th NCERT: Themes in Indian History - I

28. MERCOSUR as a sub-regional bloc excludes which of these


countries?
a) Argentina
b) Mexico
c) Brazil
d) Peru
Solution: b)

Justification: Eliminate Mexico, because MERCOSUR is a South-


American trading bloc; Mexico is a North American country. So, B is the
answer.

It was established in 1991. Its full members are Argentina, Brazil,


Paraguay and Uruguay. Its associate countries are Bolivia, Chile, Peru,
Colombia, Ecuador and Suriname.

Mercosur suspended Venezuela sometime back from it for violating the


bloc’s democratic principles and failing to meet its basic standards.

Learning: Brazil urges India to broaden MERCOSUR presence and to


expand its commercial and multilateral footprints in South America.

India and the Mercosur bloc have stepped up efforts to expand their
preferential trade agreement (PTA) to make greater inroads into the
other’s market.

Q Source: http://www.thehindubusinessline.com/economy/india-
mercosur-pact/article9455464.ece

29. The Second Administrative Reforms Commission (SARC)


describes which of these as “the master key to good governance”?
a) Building social capital
b) Establishing e-governance

2
WWW.INSIGHTSIAS.COM
7
INS INSIGHTS IAS PRELIMS TEST SERIES - 2017

c) Inculcating ethics in Governance


d) Right to information (RTI)
Solution: d)
Learning: Good governance has four elements- transparency,
accountability, predictability and participation.

• Transparency refers to availability of information to the general


public and clarity about functioning of governmental institutions.
• RTI opens up government’s records to public scrutiny, thereby
arming citizens with a vital tool to inform them about what the
government does and how effectively, thus making the government
more accountable.
• Access to information can empower the poor and the weaker
sections of society to demand and get information about public
policies.
• Without good governance, no amount of developmental schemes
can bring improvements in the quality of life of the citizens.
• It has been rightly been seen as the key to strengthening
participatory democracy and ushering in people centred
governance.
Q Source: 2nd ARC Reports

30. The new National Steel Policy (NSP) 2017 as approved by the
Cabinet provides for
1. Acquisition of state-run steel plants globally to safeguard
strategic steel imports
2. Reducing reliance on foreign investment in steel sector and
ensuring that one hundred per cent of steel consumption is
manufactured domestically
3. Ensuring domestic availability of iron ore, coking coal and
natural gas
Select the correct answer using the codes below.

a) 3 only
b) 1 and 2 only
c) 1 only

2
WWW.INSIGHTSIAS.COM
8
INS INSIGHTS IAS PRELIMS TEST SERIES - 2017

d) 2 and 3 only
Solution: a)
Justification: Statement 1: The policy seeks to create self-sufficiency
in steel production by providing policy support & guidance to private
manufacturers, MSME steel producers, CPSEs. There is no word on
strategic acquisitions. So, 1 is wrong.

Statement 2: It also talks about facilitating foreign investment to boost


steel production in India.

Statement 3: Natural gas and coal are the two primary fuels used in iron
production, hence vital for steel industry.

Q Source: PIB Releases: 03rd May 2017

31. If a bill seeks to amend the federal provisions of the Constitution


a) It must also be ratified by the legislatures of at least two-thirds
of the states by a special majority
b) It must be introduced in the Parliament with prior consent from
the President
c) It must receive the approval of a constitutional bench of
Supreme Court under Article 143 before introduction in the
Parliament
d) None of the above is correct in this regard.
Solution: d)
Justification: Option A: It should be ratified by the legislatures of half
of the states by a simple majority, that is, a majority of the members of
the House present and voting. No special majority or two-thirds states
are needed. So, A is incorrect.

Option B: After duly passed by both the Houses of Parliament and


ratified by the state legislatures, where necessary, the bill is presented to
the president for assent. No prior consent is required.

2
WWW.INSIGHTSIAS.COM
9
INS INSIGHTS IAS PRELIMS TEST SERIES - 2017

The power to initiate an amendment to the Constitution lies with the


Parliament.

Q Source: Chapter on Constitutional amendment: Indian Polity: M


Laxmikanth

32. Which one of the following statements about the ‘polar jet
streams’ is true?
a) These winds are weakest in tropopause.
b) They supply energy to the circulation of surface storms.
c) Polar fronts and polar jet streams are the same phenomena.
d) All of the above are correct.
Solution: b)
Justification: Option A and C: It is a belt of powerful upper-level winds
that sits atop the polar front. So, C is incorrect.

The winds are strongest in the tropopause, which is the upper boundary
of the troposphere, and move in a generally westerly direction in
midlatitudes. So, A is incorrect.

Option B: Jet streams significantly affect surface airfiows. When they


accelerate, divergence of air occurs at the altitude of the jet stream; this
promotes convergence near the surface and induction of cyclonic
motion. Jet streams supply energy to surface storms and direct their
path. So, B is correct.

Jet streams also cause a convergence of air aloft and subsidence near the
surface, resulting in intensification of high-pressure systems. As such, jet
streams are often described as weather makers.

Q Source: http://climate.ncsu.edu/edu/k12/.atmosphere_circulation

3
WWW.INSIGHTSIAS.COM
0
INS INSIGHTS IAS PRELIMS TEST SERIES - 2017

33. Why scientists today place high priority on Cryosphere


Studies on earth?
1. It stores most of world’s freshwater.
2. It plays a significant role in the global climate.
Which of the above is/are correct?

a) 1 only
b) 2 only
c) Both 1 and 2
d) None
Solution: c)
Justification: Statement 1 and 2: The Cryosphere is the second largest
component of the climate system, after the ocean, that stores about 75%
of the world’s freshwater.

• The Himalaya forms the most important concentration of snow


covered region outside the polar region.
• The Himalayan glaciers are highly sensitive to the on-going
warming. Some of Himalayan glaciers form the perennial source of
major rivers. Changes in glaciers are one of the clearest indicators
of alterations in regional climate, since they are governed by
changes in accumulation (from snowfall) and ablation (by melting
of ice).
• The difference between accumulation and ablation or the mass
balance is crucial to the health of a glacier.
• In terms of the ice mass and its heat capacity, therefore, it plays a
significant role in the global climate.
Q Source: http://www.moes.gov.in/programmes/cryosphere-studies-
himalaya

34. The term ‘Ecumenopolis’, sometimes seen in news, implies


a) A net-zero smart city campus
b) A single planet-wide city
3
WWW.INSIGHTSIAS.COM
1
INS INSIGHTS IAS PRELIMS TEST SERIES - 2017

c) A crime free zone


d) Satellite centres of emerging sub-urban areas
Solution: b)
Learning: The word was invented in 1967 by the Greek city planner
Constantinos Doxiadis to represent the idea that in the future urban
areas and large metropolitan areas would eventually fuse and there
would be a single continuous worldwide city.

This concept was already current in science fiction in 1942, with Trantor
in the Foundation series.

It is thus a hypothetical concept of a planetwide city or a city made of the


whole world.

Q Source: http://indianexpress.com/article/lifestyle/books/its-all-
over-beam-me-up-scotty/

http://www.thehindu.com/thread/technology/article7779855.ece

35. Which of the following is most closely related to ‘Tribal


Economy’?
a) Mining and Quarrying
b) Footloose Plantation
c) Intensive commercial cultivation
d) Shifting cultivation
Solution: d)
Learning: Shifting cultivation is considered to be the most ancient
system of agriculture dating back to the lower Neolithic period.

It is also known as "Field Forest Rotation" or slash and burn agriculture.

Shifting cultivation is practiced in some form or other in almost all the


tribal areas in some states like Orissa.

3
WWW.INSIGHTSIAS.COM
2
INS INSIGHTS IAS PRELIMS TEST SERIES - 2017

Bondo, dongaria Kondha and most primitive tribal groups (PVTGs)


practice shifting cultivation.

Since tribals do not generally indulge in commercial mining and


intensive cultivation, options A, B and C are wrong.

Q Source: 11th NCERT: Indian Economic Development

36. Consider the following statements.


1. Wind will strike your back and continue to the sea if you are
sitting on the beach facing the sea on a sunny afternoon.
2. Wind will strike your face and continue to the sea if you are
sitting on the beach with your back facing the sea on cold night.
Which of the above is/are correct?

a) 1 only
b) 2 only
c) Both 1 and 2
d) None
Solution: b)
Justification: This question is on land and sea breeze effects.

During the day, this happens:

Statement 1:

3
WWW.INSIGHTSIAS.COM
3
INS INSIGHTS IAS PRELIMS TEST SERIES - 2017

So, if you facing the sea, wind will strike your face and continue to land.

Statement 2: During the night this happens:

So, if you are facing the sea, wind will strike your back and continue to
the sea.

Q Source: 7th NCERT: Geography

3
WWW.INSIGHTSIAS.COM
4
INS INSIGHTS IAS PRELIMS TEST SERIES - 2017

37. CARE WWF Alliance focuses on tackling which of these


problems?
1. Livelihood options for vulnerable people
2. Address root causes of poverty and environmental degradation
3. Build a global network for helping in sustainable food
production
Select the correct answer using the codes below.

a) 2 only
b) 1 and 3 only
c) 3 only
d) 1, 2 and 3
Solution: d)
Justification: Statement 2 and 3: In 2008, CARE and WWF launched
a strategic alliance to address the root causes of poverty and
environmental degradation.

Through this partnership, CARE and WWF already work side by side on
many projects, envisioning local and global food systems that support
healthy ecosystems to better feed and nourish more than 7 billion people
today and a projected 9.6 billion people by 2050.

Statement 1: In a world of inequality, degradation of natural resources,


and a changing climate, the CARE WWF Alliance is focused on
integrated solutions that contribute to broader livelihood options for
vulnerable women and men, and the conservation of globally important
biodiversity.

Q Source: https://www.worldwildlife.org/partnerships/care-wwf-
alliance

3
WWW.INSIGHTSIAS.COM
5
INS INSIGHTS IAS PRELIMS TEST SERIES - 2017

38. Match the following extremist organizations, frequently seen


in news, to the countries they mainly operate in or are based in.
1. Hamas A. Gaza Strip

2. LTTE B. Nigeria

3. Hezbollah C. Sri Lanka

4. Boko Haram D. Lebanon

Select the correct answer using the codes below.

a) 1D, 2B, 3C, 4A


b) 1C, 2A, 3B, 4D
c) 1A, 2C, 3D, 4B
d) 1D, 2B, 3A, 4C
Solution: c)
Justification: Statement 1: It has been the de facto governing
authority of the Gaza Strip since its takeover of that area in 2007. During
this period it fought several wars with Israel.

Statement 2: Founded by Velupillai Prabhakaran, it waged a secessionist


nationalist insurgency to create an independent state of Tamil Eelam in
the north and east of Sri Lanka for Tamil people. This campaign led to
the Sri Lankan Civil War (1983-2009).

Statement 3: Hezbollah was conceived by Muslim clerics and funded by


Iran primarily to harass the Israeli occupation.

Its leaders were followers of Ayatollah Khomeini, and its forces were
trained and organized by Revolutionary Guards that arrived from Iran
with permission from the Syrian government, which was in occupation
of Lebanon at the time.

Statement 4: An Islamic extremist group based in north-eastern Nigeria,


also active in Chad, Niger and northern Cameroon, it was responsible for
the Nigerian civil war.

Q Source: News items


3
WWW.INSIGHTSIAS.COM
6
INS INSIGHTS IAS PRELIMS TEST SERIES - 2017

39. Consider the following about the Earth System Science


Organization (ESSO).
1. Operating as an executive arm of the Ministry of Science and
Technology, it finances centres of higher learning working on
earth research in India.
2. One of the mandates of ESSO is to promote research in polar
science of both Antarctic and Arctic Regions to understand
climatic changes.
Which of the above is/are correct?

a) 1 only
b) 2 only
c) Both 1 and 2
d) None
Solution: b)
Justification: Statement 1: It operates as an executive arm of the
Ministry of Earth Sciences (MoES) for it policies and programmes. The
ESSO provides overall direction for the centres/units and review the
implementation of the programs like:

• Centre for Marine Living Resources and Ecology (CMLRE)


• Integrated Coastal and Marine Area Management (ICMAM)
• National Centre for Seismology (NCS) etc.
Statement 2: It does so to understand the various phenomenon and
processes of these regions on global climate and weather, in particularly
on the Indian Ocean.

Cold currents from polar regions have a clear bearing on Indian


monsoon and global climate. Moreover, melting of polar ice has
consequences for global weather.

Q Source: http://www.moes.gov.in/

40. Which among the following regions functions as the most


effective ‘carbon sink’ and also known as the lungs of the earth?
3
WWW.INSIGHTSIAS.COM
7
INS INSIGHTS IAS PRELIMS TEST SERIES - 2017

a) Savanna
b) Sahara region in Africa
c) Tropical rainforests
d) Antarctica
Solution: c)
Learning: Tropical rainforests are often called the “lungs of the planet”
because they generally draw in carbon dioxide and breathe out oxygen.

The Amazon in South America is the largest, most diverse tropical


rainforest on Earth, covering an area of five and a half million square
kilometres and thus the largest forest lung of the earth.

Q Source: General Concepts: Environment

41.Consider the following about important Buddhist Heritage sites


that are under consideration for UNESCO World Heritage Site list.
1. Nagarjunakonda is known for hosting master pieces of Ikshvaku
art and architecture in the form of well sculptured ayaka-slabs
and Buddhist art.
2. Bojjannakonda is a unique Buddhist site which has exclusively
preserved Theravada Buddhism in its purest form.
Which of the above is/are correct?

a) 1 only
b) 2 only
c) Both 1 and 2
d) None
Solution: a)
Justification: Buddhist Heritage sites at Salihundam (Srikakulam
district) and Sankaram near Anakapalle in Visakhapatnam district, in
addition to Lepakshi (Anantapur district) and the Nagarjunakonda

3
WWW.INSIGHTSIAS.COM
8
INS INSIGHTS IAS PRELIMS TEST SERIES - 2017

International Museum (Guntur district) are likely to find a place in the


list of UNESCO World Heritage Sites.

Statement 1: Nagarjunakonda is a a great religious and educational


centre of Brahmanism and Buddhism.

Nagarjunkonda gets its name from the noted Buddhist scholar and
philosopher-Acharya Nagarjuna who is said to have founded the
Madhyamika school of Mahayana Buddhism which greatly influenced
the masses in Andhra Pradesh of those days.

You may read details here (not necessary however)


http://asi.nic.in/asi_museums_nagarjunakonda.asp

Statement 2: Sankaram, also known as Bojjannakonda, which was


excavated under the aegis of Alexander Rim in 1906 features all the three
phases of Buddhism: Hinayana, Mahayana and Vajrayana. So, 1 is
wrong.

Q Source: Insights Arts and Culture PT Exclusive Module 2017

42. As per the periodic Swachh Sarvekshan-2017, the cleanest


and dirtiest cities respectively are
a) Indore and Gonda
b) Mumbai and Ranikhet
c) Ranchi and Durgapur
d) Aizwal and Kishannagar
Solution: a)
Learning: It was released by Union Urban Development Minister
under Swachh Sarvekshan-2017.

Indore of MP has been adjudged as the cleanest city in India while


Gonda in Uttar Pradesh has been ranked as the dirtiest city among
surveyed.

3
WWW.INSIGHTSIAS.COM
9
INS INSIGHTS IAS PRELIMS TEST SERIES - 2017

• In total, 434 cities and towns with a population of one lakh and
above were surveyed.
• Last year survey had surveyed only 73 cities with a population of 10
lakh and above.
• Cities and towns in West Bengal did not take part in the survey.
• The criteria for selecting the cities for ranking for this year
included 45% marks for cities becoming open defecation-free; solid
waste management like sweeping, collection, transportation and
processing of garbage; education and capacity building.
• 25% marks were given for field inspection and another.
• 30% marks were given for citizen feedback.
Q Source: Swachh Sarvekshan-2017

43. These Primitive Tribal Group (PTG) of Nilgiri Hills are


famous as snake-catchers and some members were recently
recruited by Florida Wildlife Officials to hunt pythons in wetlands.
They are
a) Koras
b) Irulas
c) Bondas
d) Nirandhas
Solution: b)
Learning: Wildlife officials of Florida recruited Irula tribesmen from
Tamil Nadu to hunt the Burmese pythons believed to be decimating
native mammals in the Everglades, a wetlands preserve in the southern
tip of the U.S.

Irulas are one of the oldest tribes in India. The term Irula means being
capable of finding one's path in dark forests. They are known as the last
"forest scientists" of the world, they have been snake catchers and rat
catchers for generations.

4
WWW.INSIGHTSIAS.COM
0
INS INSIGHTS IAS PRELIMS TEST SERIES - 2017

Q Source: http://www.thehindu.com/todays-paper/tp-national/Irula-
snake-catchers-recruited-to-hunt-pythons-in-U.S.-
wetland/article17099815.ece

44. Which of the following goes against the ideal of Panchayati


Raj Institutions (PRIs) as enshrined in the constitution?
a) Delegation of financial powers to local bodies
b) Centralization of powers in the hands of bureaucracy
c) Decentralization of planning process to Gram Sabhas
d) Increased political accountability through local social audits
Solution: b)
Justification: Option B: Core values behind establishment of PRIs are
decentralization, accountability and increased delegation of function for
empowerment of people at grass roots level.

Centralization of powers in hands of civil servants goes against this very


notion of empowerment, and thus wrong.

Q Source: Chapter on PRIs: Indian Polity: M Laxmikanth

4
WWW.INSIGHTSIAS.COM
1
INS INSIGHTS IAS PRELIMS TEST SERIES - 2017

45. Consider the following statements.


1. National Mission on Cultural Mapping and Roadmap is sub-
component of Project MAUSAM intended to document India’s
strategic heritage.
2. Scheme of Buddhist & Tibetan Culture Art gives financial
assistance to the voluntary Buddhist or Tibetan organizations
engaged in Buddhist research or value propagation.
3. Scheme for Development of the Cultural Heritage of the
Himalayas gives grant to Colleges and Universities that engage
in Himalayan conservation research.
Select the correct answer using the codes below.

a) 1 only
b) 2 and 3 only
c) 3 only
d) 2 only
Solution: b)
Justification: Statement 1: It is not a part of MAUSAM.

Under this Mission, at broad-level, there are three important objectives


as follows:

• National Cultural Awareness Abhiyan:


• Nationwide Artist Talent Hunt/Scouting Programme
• National Cultural Workplace: Centralised Transactional Web
Portal with database and demography of cultural assets and
resources including all art forms and artists
Statement 2: The scheme intends to give financial assistance to the
voluntary Buddhist/Tibetan organizations including Monasteries of all-
India character engaged in the propagation and scientific development of
Buddhist/Tibetan culture, tradition and research in related fields.

Statement 3: To be eligible for grants, the Colleges and Universities


should introduce in their curricula or research course aspects of studies
relating to preservation of Himalayan art and culture.
4
WWW.INSIGHTSIAS.COM
2
INS INSIGHTS IAS PRELIMS TEST SERIES - 2017

The grants will be ad-hoc and of non-recurring nature.

Q Source: http://www.indiaculture.nic.in/national-mission-cultural-
mapping-and-roadmap

46. Albedo is a critical factor in determining the warming of


atmosphere via indirect radiation. Which among the following
materials is likely to have the lowest albedo?
a) Thick cloud
b) Fresh snow in Antarctica
c) Moist ploughed soil
d) Mirror
Solution: c)
Justification: Albedo is the fraction of solar energy (shortwave
radiation) reflected from the Earth back into space.

It is a measure of the reflectivity of the earth's surface. Ice, especially


with snow on top of it, has a high albedo: most sunlight hitting the
surface bounces back towards space.

Water is much more absorbent and less reflective. So, if there is a lot of
water, more solar radiation is absorbed by the ocean than when ice
dominates.

Soil has least reflectivity among all options A to D, and thus the most
appropriate answer.

Q Source: 11th NCERT: Fundamentals of Physical Geography

47. Which of these companies headquartered in India are Multi-


national companies today?
1. Bajaj Auto
4
WWW.INSIGHTSIAS.COM
3
INS INSIGHTS IAS PRELIMS TEST SERIES - 2017

2. Dabur
3. Bharti Entreprises
4. Café Coffee Day
Select the correct answer using the codes below.

a) 3 only
b) 1, 2 and 4 only
c) 2 and 3 only
d) 1, 2, 3 and 4
Solution: d)
Justification: Statement 1: Bajaj Auto is the world's sixth-largest
manufacturer of motorcycles and the second-largest in India. It is the
world's largest three-wheeler manufacturer.

Statement 2: Founded in 1884 by SK Burman, a physician in West


Bengal, to produce and dispense Ayurvedic medicines, it is an
international brand today.

Statement 3: Founded in 1976 by Sunil Bharti Mittal, it today operates


in 18 countries across Asia and Africa. Bharti Enterprises owns
businesses spanning across telecommunications, agri business, financial
services and manufacturing.

Statement 4: Founded in 1996 and headquartered in Bengaluru, it serves


in India, Austria, Czech Republic, Malaysia and Egypt.

Q Source: General Studies

48. Consider the following about Insolvency and Bankruptcy


Board of India established recently.
1. It is an executive body established by the Competition
Commission of India (CCI).
2. It can make model bye-laws to be adopted by insolvency
professional agencies working in India.

4
WWW.INSIGHTSIAS.COM
4
INS INSIGHTS IAS PRELIMS TEST SERIES - 2017

3. It is chaired by Union Minister of Corporate Affairs.


Select the correct answer using the codes below.

a) 1 and 3 only
b) 2 only
c) 3 only
d) None of the above
Solution: b)
Background: In India, the legal and institutional machinery for dealing
with debt default has not been in line with global standards.

• The recovery action by creditors, either through the Contract Act or


through special laws such as the Recovery of Debts Due to Banks
and Financial Institutions Act, 1993 and the Securitisation and
Reconstruction of Financial Assets and Enforcement of Security
Interest Act, 2002, has not had desired outcomes.
• So, this board established in accordance with the provisions of The
Insolvency and Bankruptcy Code, 2016 is considered as the biggest
economic reform next only to GST.
Justification: Statement 2: The objective of the new law is to promote
entrepreneurship, availability of credit, and balance the interests of all
stakeholders by consolidating and amending the laws relating to
reorganization and insolvency

For this it regulates the conduct of insolvency professional agencies


working in India.

Statement 3: Dr. M. S. Sahoo, an acclaimed thought leader in the area of


securities markets and a distinguished public servant, currently serves as
Chairperson of the Insolvency and Bankruptcy Board of India.

Q Source: http://www.mca.gov.in/

49. Rivers of the Deccan Plateau are not navigable except in their
lower course and delta region. This is mainly because

4
WWW.INSIGHTSIAS.COM
5
INS INSIGHTS IAS PRELIMS TEST SERIES - 2017

a) Rivers change courses frequently in peninsular plateau.


b) They don’t form estuaries en course.
c) The river bed is soft and prone to instability.
d) These rivers are seasonal and only rain-fed.
Solution: d)
Justification: Option A: Peninsular rivers are known to flow in short
fixed courses due to the hard terrain. So, A is wrong.

Option B: This is an irrelevant statement.

Option C: These rivers have rocky beds and highly stable. So, C is wrong.

Option D: They swell up in the monsoons and are reduced to a trickle in


winters. They are thus not navigable except in their lower course and
delta region.

Also the hard rocks in the Deccan Plateau have numerous rapids and
waterfalls so the rivers are not suited to be used as a mode of transport.

Q Source: Longman ICSE 10th Standard Geography: Chapter 2

50. The national income of a nation is best defined as the


a) Annual revenue of the government
b) Total earnings from abroad including exports
c) Surplus generated by public sector enterprises
d) Sum total of factor incomes
Solution: d)
Learning: National income measures the monetary value of the flow of
output of goods and services produced in an economy over a period of
time.

There is an income method that is used to estimate the National income.

4
WWW.INSIGHTSIAS.COM
6
INS INSIGHTS IAS PRELIMS TEST SERIES - 2017

Only those incomes that come from the production of goods and services
are included in the calculation of GDP by the income approach.

It is basically Rent+ Wages+ Profits+ Interest (earned on capital goods).


These correspond to four factors land, labour, entrepreneurship and
capital goods respectively.

Q Source: 9th NCERT: Economics

51. UJJAWALA scheme of the Government of India deals with


a) Connecting every household to the national gas pipeline grid
b) Free education for differently abled children in private schools
c) Surrender policy for Left Wing Extremism (LWE) prone areas
d) Rehabilitation and Re-integration of Victims of Trafficking and
Commercial Sexual Exploitation
Solution: d)
Learning: Objectives of the scheme are:

I. To prevent trafficking of women and children for commercial


sexual exploitation through social mobilization and involvement of
local communities, awareness programmes, workshops/seminars
and other innovative activity.
II. To facilitate rescue of victims from the place of their exploitation
and place them in safe custody.
III. To provide rehabilitation services both immediate and long-term
to the victims by providing basic amenities/needs such as shelter,
food, clothing, medical treatment including counseling, legal aid
and guidance and vocational training.
IV. To facilitate reintegration of the victims into the family and society
at large.
V. To facilitate repatriation of cross-border victims to their country of
origin.
Q Source: http://wcd.nic.in/schemes/ujjawala-comprehensive-
scheme-prevention-trafficking-and-resue-rehabilitation-and-re
4
WWW.INSIGHTSIAS.COM
7
INS INSIGHTS IAS PRELIMS TEST SERIES - 2017

52. Consider the following statements.


Assertion (A): State legislatures do not participate in the election of
Vice-President.

Reason (R): The Vice-President presides over the Council of States.

In the context of the above, which of these is correct?

a) A is correct, and R is an appropriate explanation of A.


b) A is correct, but R is not an appropriate explanation of A.
c) A is correct, but R is incorrect.
d) Both A and R are incorrect.
Solution: b)
Justification: In the constituent assembly debates, it was pointed out
that The President is the head of the State and his power extends both to
the administration by the Centre as well as to the states.

• Consequently, it is necessary that in his election, not only members


of Parliament should play their part, but the members of the state
legislatures should have a voice.
• But, when we come to the Vice-President, his normal functions are
to preside over the council of states. It is only on a rare occasion,
and that too for a temporary period, that he may be called upon to
assume the duties of the president.
That being so, it does not seem necessary that the members of the state
legislatures should also be invited to take part in the election of the Vice-
President.”

Q Source: Chapter on Vice-President: Indian Polity: M Laxmikanth

4
WWW.INSIGHTSIAS.COM
8
INS INSIGHTS IAS PRELIMS TEST SERIES - 2017

53. Consider the following statements.


1. The cold war international order was uni-polar in nature with
United States (US) as the sole superpower.
2. The US approach to the cold war was originally set out in the
Truman Doctrine which advocated disarmament and
demilitarization.
Which of the above is/are correct?

a) 1 only
b) 2 only
c) Both 1 and 2
d) None
Solution: d)
Justification: Statement 1: The world order was bi-polar with soviet
union and USA. The cold war was a rivalry between them for
domination.

Statement 2: Before the US began its expansionist policies during the


cold war, its international policy was isolationist.

It meant that the US would not intervene in the internal affairs of a


country normally unless USA is threatened or destabilised. However,
with Soviet adventurism, Truman pledged to:

• Support Free people who are resisting subjugation by armed


minorities or by outside pressures
• Commitment to a policy of containing communism not just in
Europe but throughout the world.
Q Source: 12th Contemporary World Politics NCERT

54. Consider the following about the “South Asia satellite”


recently launched by India.

4
WWW.INSIGHTSIAS.COM
9
INS INSIGHTS IAS PRELIMS TEST SERIES - 2017

1. It is a polar satellite that will provide remote sensing services to


SAARC countries.
2. The cost of the satellite has been jointly borne by all SAARC
countries.
3. It is the heaviest satellite that has been launched by India till
date.
Select the correct answer using the codes below.

a) 1 only
b) 2 and 3 only
c) 3 only
d) None of the above
Solution: d)
Justification: Statement 1: It is a geostationary satellite launched
using GSLV Mark-III (indigenous Cryogenic Upper Stage (CUS) engine).

The satellite will provide a full range of applications and services in the
fields of telecommunication and broadcasting applications, namely,
Television, Direct-to-Home (DTH), Very Small Aperture Terminals
(VSATs), Tele-education, Telemedicine and Disaster Management
Support.

Statement 2: It was borne entirely by India.

Statement 3: It weighs around four hundred Kgs. India has launched


much heavier satellites.

Learning: All SAARC countries have joined it except Pakistan. Hence,


the beneficiaries of the satellite will be Nepal, Bhutan, Bangladesh,
Afghanistan, Sri Lanka and Maldives.

• Each country can beam its own TV programmes while there will be
the possibility for a common South Asia programming
• The satellite will help the nations communicate better during
disasters and it could help establish a hotline among them.
Q Source: http://economictimes.indiatimes.com/news/science/all-
you-need-to-know-about-south-asia-
satellite/articleshow/58541606.cms

5
WWW.INSIGHTSIAS.COM
0
INS INSIGHTS IAS PRELIMS TEST SERIES - 2017

55. The term Third World, as known in history, refers to


a) Disease stricken regions in tropical countries
b) Underworld organizations operating in the Middle East
c) Colonies of European nations in Africa
d) Countries that were not aligned with either USA or Soviet Union
Solution: d)
Learning: The term has been subject to evolutionary interpretations.

Third World countries were 51labelled during the Cold War to reference
those nations that were not aligned with either the United States or the
Soviet Union.

Later, once cold war ended, it came to include many countries with
colonial pasts in Africa, Latin America, Oceania and Asia which were
poor and underdeveloped. It became a stereotype to refer to poor
countries as "third world countries".

However, recently the "Third World" term is also often taken to include
newly industrialized countries like Brazil, India and China now more
commonly referred to as part of BRIC.

Q Source: 12th Contemporary World Politics NCERT

56. Project Brahma Initiative is a community driven initiative


towards
a) Finding Universal truth
b) Building rural communities in North-eastern India
c) Building India's biodiversity knowledge base
d) Sourcing detailed street maps for crowded urban areas
Solution: c)

5
WWW.INSIGHTSIAS.COM
1
INS INSIGHTS IAS PRELIMS TEST SERIES - 2017

Learning: The Biodiversity of India (BOI) website, part of the Project


Brahma Initiative, is an open-source, community driven project, much
like Wikipedia, where anyone - regardless of their religion, nationality,
language, expertise - can contribute their knowledge of India's
biodiversity.

It aims to highlight the intricate associations between Indian biodiversity


and local cultural practices & mythology.

It will also create and compile tools such as online learning modules to
educate the public about biodiversity.

Q Source:
http://www.biodiversityofindia.org/index.php?title=Introduction_to_th
e_Brahma_Database#Biodiversity_of_India

57. Consider the following statements.


Assertion (A): More than 50% of land area in India is undergoing
desertification.

Reason (R): India does not have a specifically targeted policy or


legislative framework for combating desertification.

In the context of the above, which of these is correct?

a) A is correct, and R is an appropriate explanation of A.


b) A is correct, but R is not an appropriate explanation of A.
c) A is incorrect, but R is correct.
d) A is correct, but R is incorrect.
Solution: c)
Justification: India has about 32 % of its land under degradation and
25 % undergoing desertification. So, A is incorrect.

• Though India does not have a specific policy or legislative


framework for combating desertification as such, the concern for
5
WWW.INSIGHTSIAS.COM
2
INS INSIGHTS IAS PRELIMS TEST SERIES - 2017

arresting and reversing land degradation and desertification gets


reflected in many of our national policies.
• Some examples are National Water Policy 2012; National Forest
Policy 1988; National Agricultural Policy 2000; Forest
(Conservation) Act 1980; Environment (Protection) Act 1986;
National Environmental Policy 2006; National Policy for Farmers
2007 etc.
• The first five year plan (1951-1956) had ‘land rehabilitation’ as one
of the thrust areas. In the subsequent plans too, high priority has
been consistently attached to development of the drylands.
Q Source: MoEF Website

58. The state/union territory that has attracted the highest


Foreign Direct Investment (FDI) in the past couple of years is
a) Delhi
b) Kerala
c) Madhya Pradesh
d) Uttar Pradesh
Solution: a)
Learning: These two tables are key takeaways in the FDI trend in India.

5
WWW.INSIGHTSIAS.COM
3
INS INSIGHTS IAS PRELIMS TEST SERIES - 2017

Delhi, Haryana, Maharashtra, Karnataka, Tamil Nadu, Gujarat and


Andhra Pradesh have together attracted more than 70 per cent of total
FDI inflows to India during the last 15 years.

However, states with vast natural resources like Jharkhand, Bihar,


Madhya Pradesh, Chhattisgarh and Odisha have not been able to attract
foreign funds directly for investment in different sectors.

Q Source: Economic Survey 2015-16

59. Biological weathering will be least effective in which of these


regions?

5
WWW.INSIGHTSIAS.COM
4
INS INSIGHTS IAS PRELIMS TEST SERIES - 2017

a) Tropical rainforests
b) Arid regions or deserts
c) Coastal region
d) Temperate forests
Solution: b)
Justification: For biological weathering, high temperature and high
humidity are pre-requisites as this is vital for microbial action.

In tropical rainforests, OPTION A, there is high humidity as well as year


round warm temperatures, so biological weathering will be dominant.

Same is true for coastal regions. Temperate forests have moderate


temperature and humidity but still they will perform better than arid
regions where there is acute scarcity of moisture. So, B is most
appropriate option.

Q Source: 11th NCERT: Fundamentals of Physical Geography

60. Wiseman-Peacock hypothesis, sometimes heard among


policymakers relates to
a) Density of semi-conductors on a chip
b) Patterns of public expenditure
c) Fall of an authoritarian democratic state
d) Movement of tectonic plates in oceans
Solution: b)
Learning: There are two main ideas studying public expenditure.

• Wagner’s law states that for any country the public expenditure
rises constantly.
• Peacock and Wiseman conducted a new study based on Wagner's
law. They confirmed the validity of Wagner’s law.

5
WWW.INSIGHTSIAS.COM
5
INS INSIGHTS IAS PRELIMS TEST SERIES - 2017

• According to their hypothesis, the increase in public expenditure


doesn't follow any smooth and continuous trend but the increase in
public expenditure occurred in step like manner.
• With the kinky rise in the public expenditure it is the central
government that comes to fulfil larger and larger state activities
leaving lesser responsibilities to the regional and local public
authorities.
Q Source: Peripheral questions: Public Finance

61.Consider the following with respect to the about regional new years
celebrated in India.
1. Kutchi is celebrated in Gujarat usually in the month of March.
2. Cheti Chand is celebrated in Telangana generally in the month
of July.
3. Sajibu Cheiraoba is celebrated in Manipur in the month of
March or April.
4. Gudi Padwa is celebrated in Madhya Pradesh in the month of
January.
Select the correct answer using the codes below.

a) 2 and 3 only
b) 1 and 4 only
c) 3 only
d) 1, 2, 3 and 4
Solution: c)
Justification: Statement 1: The Kutchi people celebrate Kutchi New
Year on Ashadi Beej, that is 2nd day of Shukla paksha of Aashaadha
month of Hindu calendar. Hindu calendar month of Aashaadh usually
begins on 22 June and ending on 22 July.

Statement 2: It is Sindhi new year usually celebrated in March or April.

Statement 3: It is the lunar new year festival of the people who follow
the sanamahism religion of the Indian state of Manipur. It usually falls
during the month of March/April.
5
WWW.INSIGHTSIAS.COM
6
INS INSIGHTS IAS PRELIMS TEST SERIES - 2017

Statement 4: Gudhi Padva is a spring-time festival that marks the


traditional new year for Marathi Hindus. It is celebrated in and near
Maharashtra on the first day of the Chaitra month in March.

Q Source: Regional Culture: India

62. The time period of a simple metal pendulum will NOT


depend on
a) Major temperature variations around the pendulum
b) Mass of pendulum
c) Any change in the mass of earth
d) Length of pendulum
Solution: b)
Justification: Time period of a pendulum is T = 2Pi root of (l/g).

So, the period for a simple pendulum does not depend on the mass or the
initial angular displacement, but depends only on the length L of the
string and the value of the gravitational field strength g.

Option A: Temperature changes will increase or decrease metal length,


and thus the period of the pendulum.

Q Source: 10th Science NCERT

63. Consider the following statements with reference to the


Indus Valley civilization time periods.
1. The Ahar culture which flourished in Kerala of present day India
was famous for its blade tool industry.
2. Navdatoli near Narmada River was an important settlement
point for Malwa culture in Central India.
Which of the above is/are correct?

5
WWW.INSIGHTSIAS.COM
7
INS INSIGHTS IAS PRELIMS TEST SERIES - 2017

a) 1 only
b) 2 only
c) Both 1 and 2
d) None
Solution: b)
Justification: Statement 1: It is a Chalcolithic archaeological culture
of southeastern Rajasthan state in India, lasting from c. 3000 to 1500
BCE, contemporary and adjacent to the Indus Valley Civilization.

Situated along the Banas and Berach Rivers, as well as the Ahar River,
the Ahar-Banas people were exploiting the copper ores of the Aravalli
Range to make axes and other artefacts.

Statement 2: The Ahar culture phase was followed by the Malwa culture.
Navdatoli (west Nimar district), on the southern banks of the Narmada,
is the largest settlement of this culture.

Calibrated dates for the beginning of the settlement are in the range of
2000—1750 BCE.

Malwa ware is exceptionally rich in forms and designs. A lot of stone


artefacts, as compared to copper, were found at this culture’s site
indicating that there might have been a shortage of copper.

Q Source: Upinder Singh: A History of Ancient and Early Medieval


India: From the Stone Age to the 12th Century

64. Low productivity for agriculture in India CANNOT be


attributed to
a) Heavy dependence on monsoon
b) Lack of gross capital formation in agriculture
c) Very small percentage of net sown area compared to its
geographical area
d) Poor modern agricultural know-how in farmers

5
WWW.INSIGHTSIAS.COM
8
INS INSIGHTS IAS PRELIMS TEST SERIES - 2017

Solution: c)
Justification: Option C: Over 50% of our area is cultivated, so C can’t
be correct.

Option B: Reports suggest that GCF in agriculture is below global


average which implies that our agriculture relies on old mechanical
techniques, and thus has low productivity.

Option A: Rainfed farms, as much as 55%, will have lower productivity


than irrigated farms.

Q Source: 11th NCERT: Indian Economic Development

65. Which of these birds found in India is/are Critically


Endangered and also traded for food or taxidermy?
1. Spoon-billed Sandpiper
2. White-rumped Vulture
3. Greater Adjutant
4. Saker Falcon
Select the correct answer using the codes below.

a) 2 and 3 only
b) 1 and 4 only
c) 1 and 2 only
d) 1, 2, 3 and 4
Solution: c)
Justification: A table of some of the Threatened Birds of India in
Trade:

Sr Common Name IUC Used in trade Status in Trade


. N as/for
Cate
gory

5
WWW.INSIGHTSIAS.COM
9
INS INSIGHTS IAS PRELIMS TEST SERIES - 2017

1 Spoon-billed Sandpiper CR Meat, trade Very rarely caught


Eurynorhynchus
pygmeus

2 White-rumped Vulture CR Taxidermy, Zoo Very rarely caught


Gyps bengalensis Trade, Black now. Formerly
Magic, Meat caught
occasionally for
food and
taxidermy

3 Red-headed Vulture CR Taxidermy, Zoo Rarely caught


Sarcogyps calvus Trade

4 Indian Vulture Gyps CR Taxidermy, Zoo Very Rare


indicus Trade, Meat,
Black Magic
5 Green Peafowl Pavo EN Meat, trade, Very rarely caught
muticus Aviculture

6 Black-bellied Tern EN Meat, trade Very rarely caught


Sterna acuticauda

7 Greater Adjutant EN Zoo, Trade, Very rarely caught


Leptoptilos dubius Meat, Black
Magic
8 Egyptian Vulture EN Zoo, Trade, Rarely caught
Neophron percnopterus Release Trade

9 Saker Falcon Falco EN Falconry Rarely caught


cherrug

10 Swamp Francolin VU Meat, trade, Occasionally


Francolinus gularis Aviculture, Zoo caught
Trade, Bird
Fights

Q Source: ENVIS Portal: Avian Biodiversity

6
WWW.INSIGHTSIAS.COM
0
INS INSIGHTS IAS PRELIMS TEST SERIES - 2017

66. Politics-Administration dichotomy implies that


a) Bureaucracy should be politically active and allowed to
participate in elections.
b) Policy making and policy execution are separate activities.
c) Government should not venture out in provision of public
goods.
d) A welfare state cannot provide good governance.
Solution: b)
Justification: The politics-administration dichotomy is an important
concept in the field of public administration because it deals with the
policy-makers role as an administrator and the balancing act that is the
relationship between politics and administration.

There are two views on this concept.

• Those who support it say that politics and administration should


be distinct, claim that it will ensure an efficient, effective and
neutral bureaucracy. A politically motivated bureaucracy will not
augur well for administration. So, option A is wrong. However, the
extent of such distinction had not been clearly stated.
• The other group who argue that the two disciplines should not be
separated rest their thesis on the interconnection between politics
and administration. To them, politics and administration are
complementary.
Q Source: Basics of Public Administration

67. The salient features of the Indian Independence Act, 1947


were
1. The Act established the republics of India and Pakistan.
2. It appointed a Boundary commission to demarcate the
bordering provinces of India and Pakistan.
3. It provided for the composition of the assembly that later
framed India’s constitution.
Select the correct answer using the codes below.

6
WWW.INSIGHTSIAS.COM
1
INS INSIGHTS IAS PRELIMS TEST SERIES - 2017

a) 1 only
b) 2 only
c) 2 and 3 only
d) 3 only
Solution: b)
Justification: Statement 1: It mentioned that, “the British
Government would transfer all powers the two Dominions (NOT
republics) of India and Pakistan. So,1 is wrong.

Statement 2: It said that a Boundary Commission would demarcate the


boundaries of the provinces of the Punjab and Bengal.

The Radcliff Boundary Commission drew the boundary line separating


India and Pakistan.

Statement 3: The Act provided for the transfer of power to the


Constituent Assemblies of the two Dominions, which will have full
authority to frame their respective Constitutions. They were already
established under the Cabinet Mission Plan 1946. So, 3 is wrong.

Q Source: 12th Tamil Nadu Textbook History

68. Investment is said to have both supply side and demand side
effects. What are these?
1. Supply side effect means investment generates capital goods for
the economy that help in increasing its productive capacity.
2. Demand side effect means investment generates employment
which generates income and thus demand for goods in the
economy.
Which of the above is/are correct?

a) 1 only
b) 2 only
c) Both 1 and 2
d) None
Solution: c)

6
WWW.INSIGHTSIAS.COM
2
INS INSIGHTS IAS PRELIMS TEST SERIES - 2017

Justification: The above is based on Domar model of economic


growth. But, you can solve this question by applying common economic
logic.

He lays emphasis on the dual character of investment.

Firstly, it creates income, and secondly, it augments the productive


capacity of the economy by increasing its capital stock and generate
economic growth through the increase in production of goods and
services.

The former may be regarded as the ‘demand effect’ and the latter the
‘supply effect’ of investment. Both of these combine to make investment
a very potent factor for determining the level of economic activity.

Q Source: Fundamentals of macroeconomics

69. Consider the following mountain passes and the states they
are located in.
1. Nathula A. Himachal Pradesh

2. Nitipass B. Arunachal Pradesh

3. Shipki La C. Sikkim

4. Bomdi La D. Uttarakhand

Select the correct answer using the codes below.

a) 1D, 2A, 3B, 4C


b) 1C, 2D, 3A, 4B
c) 1A, 2B, 3C, 4D
d) 1A, 2C, 3D, 4B
Solution: b)

6
WWW.INSIGHTSIAS.COM
3
INS INSIGHTS IAS PRELIMS TEST SERIES - 2017

Justification: Statement 1: It connects the Indian state of Sikkim with


China's Tibet Autonomous Region. The pass, at around 14,140 ft above
mean sea level, forms a part of an offshoot of the ancient Silk Road.

Statement 2: It is in Uttarakhand, and the road to Kailash and


Mansarovar pass through it.

Statement 3: It lies in HP. The road to Shimla goes through this pass.
Satluj river also flows through this pass.

Statement 4: It lies in Arunachal Pradesh. The Eaglenest Wildlife


Sanctuary is near Bomdila.

Q Source: Longman ICSE 10th Standard Geography: Chapter 2

70. Cabinet Committee on Appointments is headed by


a) President of India
b) Prime Minister
c) Union Minister of Home affairs
d) Union Minister of Finance
Solution: b)
Learning: The Appointments Committee of the Cabinet (ACC) decides
appointments to several top posts under the Government of India.

• The committee is composed only of the Prime Minister of India


(who is the Chairman), and the Minister of Home Affairs.
• The Establishment Officer’s Division (EO Division) processes all
proposals for senior appointments in the Government of India that
require approval of the Appointments Committee of the Cabinet.
• The Establishment Officer (EO) is the ex-officio Member Secretary
of the Civil Services Board that is chaired by the Cabinet Secretary.
Q Source:
http://cabsec.nic.in/files/allocation/cabinet_committees.pdf

6
WWW.INSIGHTSIAS.COM
4
INS INSIGHTS IAS PRELIMS TEST SERIES - 2017

71. Vikramasenavijaya is a famous work on


a) The conquests of Emperor Vikramaditya in Patliputra
b) Poetry written during the reign of Rashtrakutas
c) A collection of strategic military doctrines of Northern Indian
empires
d) Folklore praising deeds of courage and wisdom in the times of
pandyas
Solution: b)
Learning: The Kannada literature saw its beginning during the period
of the Rashtrakutas.

• Amogavarsha’s Kavirajamarga was the first poetic work in


Kannada language.
• Pampa was the greatest of the Kannada poets. His famous work
was Vikramasenavijaya.
• Ponna was another famous Kannada poet and he wrote
Santipurana.
• The Rashtrakutas widely patronized the Sanskrit literature. There
were many scholars in the Rashtrakuta court.
• Trivikrama wrote Nalachampu and the Kavirahasya was composed
by Halayudha during the reign of Krishna III.
• The Jain literature flourished under the patronage of the
Rashtrakutas.
Q Source: 11th Tamil Nadu History Textbook

72. Consider the following statements.


1. A non-citizen of India cannot become a Member of Parliament.

6
WWW.INSIGHTSIAS.COM
5
INS INSIGHTS IAS PRELIMS TEST SERIES - 2017

2. A person who does not have his/her name on the electoral rolls
cannot contest for election to the Parliament.
Which of the above is/are correct?

a) 1 only
b) 2 only
c) Both 1 and 2
d) None
Solution: c)
Justification: Statement 1 and 2: Qualifications for Membership of Lok
Sabha are the following:

• She should be a citizen of India; So, 1 is correct.


• She should not be less than 25 years of age;
• She should not be a proclaimed criminal (i.e. he/she should not be
a convict, a confirmed debtor or otherwise disqualified by law);
• She should have his/her name in the electoral rolls in any port of
the country (he/she may not belong to the State from which he/she
contests the election); so, 2 is correct as well.
• The candidate should not be a direct beneficiary of the
Government and should not hold on office of profit under the
Government.
Q Source: 11th NCERT: Indian Constitution at Work

73. The latest country to join the United Nations as a member is


a) New Zealand
b) Finland
c) South Sudan
d) Palestine
Solution: c)
Justification: Option D: Palestine has joined as an observer at the UN,
not as a full member.

6
WWW.INSIGHTSIAS.COM
6
INS INSIGHTS IAS PRELIMS TEST SERIES - 2017

However, in 2011, the General Assembly of UNESCO voted to admit


Palestine as a member, becoming the first UN agency to admit Palestine
as a full member.

Option C: South Sudan’s independence from the rest of Sudan as a result


of the 2011 referendum that ended the decades-long civil war between
the North and the South led to its UN membership.

Learning: Membership in the United Nations is open to all peace-


loving states which accept the obligations contained in the present
Charter and, in the judgement of the Organization, are able and willing
to carry out these obligations.

• The admission of any such state to membership in the United


Nations is effected by a decision of the General Assembly upon the
recommendation of the Security Council.
• A recommendation for admission from the Security Council
requires affirmative votes from at least nine of the council's fifteen
members, with none of the five permanent members using their
veto power.
• The Security Council's recommendation must then be approved in
the General Assembly by a two-thirds majority vote.
• In principle, only sovereign states can become UN members, and
currently all UN members are sovereign states.
Q Source: UN Website:
http://www.un.org/apps/news/story.asp?NewsID=39034#.WQ827YiG
M2w

74. Which of these rays from the electromagnetic spectrum has


the highest wavelength?
a) Ultraviolet
b) Gamma rays
c) X-rays
d) Infrared rays

6
WWW.INSIGHTSIAS.COM
7
INS INSIGHTS IAS PRELIMS TEST SERIES - 2017

Solution: d)
Justification: In order of decreasing frequency (and increasing
wavelength), the various regions of the electromagnetic spectrum are:
gamma rays, x-rays, ultraviolet, visible light, infrared, microwaves, and
radio waves.

Electromagnetic energy from the sun consists mostly of a small amount


of ultraviolet, all visible light, and some infrared.

Gamma rays are used in Radiation therapy; X rays in radiation therapy


and diagnosis.

Q Source: 10th Science NCERT

75. Wetlands support human well-being in a number of ways.


Which of these is/are such contribution(s) of wetlands?
1. They cleanse polluted water in vicinity.
2. They help stabilize water supplies and protect shorelines.
3. As an ecosystem, they support a rich biodiversity.
Select the correct answer using the codes below.

a) 2 only
b) 1 and 3 only
c) 1 only
d) 1, 2 and 3
Solution: d)
Justification: Statement 2: Wetlands are our natural buffers against
increasing risk of floods, droughts and tropical cyclones. Wetlands can
6
WWW.INSIGHTSIAS.COM
8
INS INSIGHTS IAS PRELIMS TEST SERIES - 2017

act as sponges, storing peak rainfall and releasing water gradually during
lean season.

Statement 1: As ‘kidneys of landscape’, wetlands receive flows of water


and waste from upstream sources. They help stabilize water supplies,
cleanse polluted waters, protect shorelines and recharge groundwater
aquifers.

Statement 3: The extensive food chain and biological diversity in


wetlands make them ‘biological supermarkets’ due to them being
nutrient rich and supporting a wide variety of communities.

The floods in Kashmir Valley in September, 2014 and Chennai city in


December, 2015 are reminders of the ways wetland destruction can make
lives vulnerable.

Q Source: Concepts in Ecology

76. The Cartagena Declaration, 1984, is related to the


a) Protection of refugees
b) Educational rights for minorities
c) Livelihood for migrant workers
d) Crèche facilities for working women
Solution: a)
Learning: The Cartagena Declaration on Refugees was adopted by the
Colloquium on the International Protection of Refugees in Central
America, Mexico and Panama in 1984.

The declaration is a non-binding agreement but has been incorporated in


refugee law in various countries. Its principles are based on the 1951 UN
Refugee Convention and the 1967 Protocol.

The Declaration reaffirms the importance of the right to asylum, the


principle of non-refoulment and the importance of finding durable
solutions.
6
WWW.INSIGHTSIAS.COM
9
INS INSIGHTS IAS PRELIMS TEST SERIES - 2017

Q Source: https://www.hrw.org/news/2017/04/18/venezuela-
humanitarian-crisis-spilling-brazil

77.‘Abolition of Untouchability’ is a component of which of the


following categories of Fundamental Rights under the
constitution?
a) Right against Exploitation
b) Right to Equality
c) Right to Freedom
d) Right to Religion
Solution: b)
Learning: Right to equality is an important right provided for in
Articles 14, 15, 16, 17 and 18 of the constitution. It is the principal
foundation of all other rights and liberties

Article 17 of the constitution abolishes the practice of untouchability.


Practice of untouchability is an offence and anyone doing so is
punishable by law under Protection of Civil Rights Act.

Q Source: 11th NCERT: Indian Constitution at Work

78. Which among the following are components of multi lateral


trading principles as laid out by the World Trade Organization
(WTO)?
1. National Treatment
2. Reciprocity
7
WWW.INSIGHTSIAS.COM
0
INS INSIGHTS IAS PRELIMS TEST SERIES - 2017

3. Protectionism
4. Transparency in trade regulations
Select the correct answer using the codes below.

a) 1, 2 and 4 only
b) 3 and 4 only
c) 1 and 2 only
d) 2, 3 and 4 only
Solution: a)
Justification: Statement 1: Non-discrimination is the pillar behind
this rule. It has two major components: the most favoured nation (MFN)
rule, and the national treatment policy.

The MFN rule requires that a WTO member must apply the same
conditions on all trade with other WTO members.

National treatment means that imported goods should be treated no less


favourably than domestically produced goods.

Statement 2: For a nation to negotiate, it is necessary that the gain from


doing so be greater than the gain available from unilateral liberalization;
reciprocal concessions intend to ensure that such gains will materialise

Statement 3: This is the opposite of openness that WTO advocates.

Statement 4: The WTO members are required to publish their trade


regulations, to maintain institutions allowing for the review of
administrative decisions affecting trade, to respond to requests for
information by other members, and to notify changes in trade policies to
the WTO.

Q Source:
https://www.wto.org/english/thewto_e/whatis_e/tif_e/fact2_e.htm

7
WWW.INSIGHTSIAS.COM
1
INS INSIGHTS IAS PRELIMS TEST SERIES - 2017

79. The British government recognized the “Right of Dominion”


for India for the first time under
a) August Offer, 1940
b) Queen’s Proclamation, 1858
c) Cripps Mission, 1942
d) Montague Declaration, 1917
Solution: c)
Learning: Under the Cripps Mission, Indians were given full liberty to
frame a constitution for themselves after fulfilling certain British
obligations.

The Viceroy was to discuss with Indian leaders in deciding the principles
on which the new constitution was to be based.

For the first time an amount of mist was reposed in the Indians.

The scheme was calculated to please the Congress, Indian states and
Muslim league at the same time.

Q Source: 10th NCERT: Contemporary India - II

80. Blood rain that had been observed for some time in Kerala is
generally caused due to
a) Presence of laterite soil in the region
b) Excess of nitric and sulphur oxides in the atmosphere
c) Specific type of green species of microalgae in the air
d) Dispersal of iron oxides in the air
Solution: c)
Learning: Since 1896, reports have been coming in of sporadic
instances of red coloured rain over parts of Kerala and Sri Lanka.

The latest one was in 2012 over Kerala.

7
WWW.INSIGHTSIAS.COM
2
INS INSIGHTS IAS PRELIMS TEST SERIES - 2017

It was initially thought that the rains were coloured by fallout from a
hypothetical meteor burst.

But a study commissioned by the Government of India concluded that


the rains had been coloured by airborne spores from a locally prolific
terrestrial green alga from the genus Trentepohlia. The exact species was
later identified as T. annulata.

Q Source: http://www.thehindu.com/sci-tech/science/unravelling-the-
blood-rain-mystery/article7057859.ece

http://www.bbc.com/news/magazine-20028490

81. The Joint Lender’s Forum (JLF), recently seen in news, was
conceived to tackle
a) Stressed assets
b) Disaster management funding
c) Sovereign debt
d) International taxation disputes
Solution: a)
Learning: Under the stressed asset norms of RBI that took effect in
2014, as soon as interest payments on a loan are delayed by 60 days, a
JLF comprising all lenders must be put in place.

And within 45 days, the JLF must come up with a corrective action plan
(CAP) and decide whether the debtor merely needs some hand-holding,
or if the forum should opt for debt restructuring or recovery.

The Reserve Bank of India (RBI) has tightened the rules around making
the Joint Lenders’ Forum (JLF) more effective, directing banks not to
break any rules and to meet all deadlines.

Q Source: http://www.financialexpress.com/banking-finance/rbi-
tightens-jlf-rules-directs-banks-not-to-break-any-rules-threatens-to-
impose-penalties/656416/
7
WWW.INSIGHTSIAS.COM
3
INS INSIGHTS IAS PRELIMS TEST SERIES - 2017

82. Consider the following statements.


Assertion (A): Stem cells of one person cannot be transferred to
another person.

Reason (R): Each individual carries a unique DNA that results


from genetic inheritance.

In the context of the above, which of these is correct?

a) A is correct, and R is an appropriate explanation of A.


b) A is correct, but R is not an appropriate explanation of A.
c) A is incorrect, but R is correct.
d) A is correct, but R is incorrect.
Solution: c)
Justification: There are two types of stem cell transplantation.

Under Autologous stem-cell transplantation, stem cells are removed


from a person, stored, and later given back to that same person.

Under allogenic stem cell transplantation donor and the recipient of the
stem cells are different people.

An autologous stem cell transplant's goal is to restore the body's ability


to make normal blood cells after high-dose chemotherapy or radiation.

Learning: Stem cell is an undifferentiated cell of a multicellular


organism which is capable of giving rise to indefinitely more cells of the
same type and from which certain other kinds of cell may be formed by
the cellular differentiation.

In the stem cell treatments new adult cells are introduced into the
damaged tissue to treat the disease.

7
WWW.INSIGHTSIAS.COM
4
INS INSIGHTS IAS PRELIMS TEST SERIES - 2017

Q Source: http://www.thehindu.com/news/cities/Vijayawada/Stem-
Cell-Therapy-camps-in-city-and-Hyderabad/article14414552.ece

83. Which of these species are listed in Schedule I of the Wildlife


Protection Act, 1972?
a) Andaman teal, Jerdon's Baza, Bengal florican
b) Nilgai, Sambar, Sponges
c) Muntjac, Goral, Chital
d) Hedge Dog, Indian Porupine, Five-striped squirrel
Solution: a)
Learning: Schedule I and part II of Schedule II provide absolute
protection - offences under these are prescribed the highest penalties.

Species listed in Schedule III and Schedule IV are also protected, but the
penalties are much lower.

Schedule V includes the animals which may be hunted. The plants in


Schedule VI are prohibited from cultivation and planting.

You can see the schedules here


http://envfor.nic.in/legis/wildlife/wildlife1.html (End of the page)

Q Source:
http://wiienvis.nic.in/Database/ScheduleSpeciesDatabase_7969.aspx

84. According to Wegner, which among the following forces


could be said responsible for continental drift?
a) Ridge push force
b) Convection currents
c) Magma Force
7
WWW.INSIGHTSIAS.COM
5
INS INSIGHTS IAS PRELIMS TEST SERIES - 2017

d) Tidal Force
Solution: d)
Justification: According to Wegener, all the continents formed a single
continental mass, a mega ocean surrounded by the same.

• He argued that, around 200 million years ago, the super continent,
Pangaea, began to split.
• Wegener suggested that the movement responsible for the drifting
of the continents was caused by pole-fleeing force and tidal force.
• The polar-fleeing force relates to the rotation of the earth.
• The second force that was suggested by Wegener—the tidal force—
is due to the attraction of the moon and the sun that develops tides
in oceanic waters.
• Wegener believed that these forces would become effective when
applied over many million years.
Q Source: 11th NCERT: Fundamentals of Physical Geography

85. The proposals for re-introduction of once extinct Cheetah in


India have been put on hold because
1. There are financial difficulties in the resettlement and
rehabilitation of communities that will be affected by this re-
introduction.
2. The Supreme Court had stayed the re-introduction on ecological
grounds.
Which of the above is/are correct?

a) 1 only
b) 2 only
c) Both 1 and 2
d) None
Solution: c)

7
WWW.INSIGHTSIAS.COM
6
INS INSIGHTS IAS PRELIMS TEST SERIES - 2017

Justification: Statement 1: Reintroduction of the cheetah to Madhya


Pradesh's Nauradehi Wildlife Sanctuary requires financing the relocation
of 90-odd villages out of the park's area in Sagar district for which MP
government and Centre are at odds with each other.

The state government believes that Centre should foot the bill, whereas
Centre thinks otherwise.

Statement 2: The 2012 judgment of SC rested on the fact that, “Scientific


studies show that the African Cheetahs and Asian Cheetahs are different,
both genetically and also in their characteristics and the reintroduction
of Cheetah was also against the International Union for the Conservation
of Nature (IUCN) guidelines on translocation of exotic wildlife species.”

Q Source: http://www.moef.nic.in/downloads/public-
information/ProjectCheetahBrochure.pdf

http://indiatoday.intoday.in/story/india-cheetah-reintroduction-
nauradehi-wildlife-sanctuary-madhya-pradesh/1/340619.html

86. The programmes of the Non-Cooperation Movement 1919


involved
1. Surrender of titles and honorary positions
2. Resignation of membership from the local bodies
3. Disobeying all British laws
4. Boycott of foreign goods
Select the correct answer using the codes below.

a) 2 and 4 only
b) 1 and 3 only
c) 1, 2 and 4 only
d) 2 and 3 only
Solution: c)

7
WWW.INSIGHTSIAS.COM
7
INS INSIGHTS IAS PRELIMS TEST SERIES - 2017

Justification: Statement 3: This was a part of the Civil Disobedience


movement.

Other programmes were:

• Boycott of elections held under the provisions of the 1919 Act.


• Boycott of government functions, courts, government schools and
colleges.
• Establishment of national schools, colleges and private panchayat
courts.
• Popularizing swadeshi goods and khadi
The movement began with Mahatma Gandhi renouncing the titles, which
were given by the British. Other leaders and influential persons also
followed him by surrendering their honorary posts and titles.

Q Source: 12th Tamil Nadu History Textbook

87. The World Conservation Monitoring Centre (WCMC) is an


executive agency of
a) Ministry of Environment, Forest and Climate Change
(MoEFCC), Government of India
b) World Wild Life Fund (WWF)
c) International Union for Conservation of Nature (IUCN)
d) United Nations Environment Programme (UNEP)
Solution: d)
Learning: The activities of UNEP-WCMC include biodiversity
assessment, support to international conventions such as the CBD and
the CITES, capacity building and management of both aspatial and
spatial data on species and habitats of conservation concern.

UNEP-WCMC has a mandate to facilitate the delivery of the global


indicators under the CBD's 2010 Biodiversity Target on the rate of loss of
biological diversity, and works alongside the CITES Secretariat
producing a range of reports and databases.
7
WWW.INSIGHTSIAS.COM
8
INS INSIGHTS IAS PRELIMS TEST SERIES - 2017

It also manages the World Database of Protected Areas in collaboration


with the IUCN World Commission on Protected Areas.

Q Source: https://www.unep-wcmc.org/about-us

88. Which of these inert gases is used as a “breathing gas” and


substituted for nitrogen in the breathing air used by deep sea
divers?
a) Krypton
b) Neon
c) Argon
d) Helium
Solution: d)
Justification: Most breathing gases are a mixture of oxygen and one or
more inert gases.

A variety of breathing gases have been developed to improve on the


performance of ordinary air by reducing the risk of decompression
sickness (when dissolved gases come out of solution in bubbles and can
affect just about any body area including joints).

So, a safe breathing gas must not become toxic when being breathed at
high pressure such as when underwater.

Learning: Oxygen and nitrogen are examples of gases that become


toxic under pressure.

• Trimix is a mixture of oxygen, nitrogen and helium and is often


used at depth in technical diving and commercial diving instead of
air to reduce nitrogen narcosis and to avoid the dangers of oxygen
toxicity.

7
WWW.INSIGHTSIAS.COM
9
INS INSIGHTS IAS PRELIMS TEST SERIES - 2017

• Heliox is a mixture of oxygen and helium and is often used in the


deep phase of a commercial deep dive to eliminate nitrogen
narcosis.
Q Source: General Science questions

89. Consider the following statements about literary sources in


Ancient India.
1. The accounts of Ratnavali and Nagananda provide useful
information about the kingdom of Harsha.
2. Jatakas provide information about the Mauryas.
3. The Sangam Tamil literature mentions about the Mauryan
invasion of the far south.
Select the correct answer using the codes below.

a) 1 only
b) 3 only
c) 1 and 2 only
d) 1, 2 and 3
Solution: d)
Justification: Statement 1: The chief sources for tracing the history of
Harsha and his times are the Harshacharita written by Bana and the
Travel accounts of Hiuen Tsang. Bana was the court poet of Harsha.

• Besides these two sources, the dramas written by Harsha, namely


Ratnavali, Nagananda and Priyardarsika also provide useful
information.
• The Banskhera inscription contains the signature of Harsha.
Statement 2 and 3: The Puranas and the Buddhist literature such as
Jatakas provide information on the Mauryas. The Ceylonese Chronicles
Dipavamsa and Mahavamsa throw light on the role Asoka in spreading
Buddhism in Sri Lanka.

Taranatha, the Tibetan monk states that Bindusara conquered 16 states


comprising ‘the land between the two seas’. The Sangam Tamil literature
also confirms the Mauryan invasion of the far south.

8
WWW.INSIGHTSIAS.COM
0
INS INSIGHTS IAS PRELIMS TEST SERIES - 2017

Q Source: 11th Tamil Nadu History Textbook

90. Foundation for Ecological Security (FES) is a/an


a) A unit of Oxfam working on livelihood security in conflict prone
areas
b) Offshoot of United Nations Environment Programme (UNEP)
working in developing countries
c) Non-profit organisation working on environmental causes and
based in Gujarat
d) Attached office of the Ministry of Rural Development (MoRD)
working in advisory capacity
Solution: c)
Learning: It was set up in 2001 to reinforce the massive and critical
task of ecological restoration in the country.

The three fundamental dimensions or cornerstones of FES are:


Ecological Restoration; building Commons and Community Institutions
and ensuring Rural Livelihoods.

It collaborates with Panchayat Raj and other democratic village


institutions, as well as appropriate civil society organisations, in their
efforts to fulfil the objectives of the society, and to provide technical and
financial assistance to them.

Q Source: http://www.indianbiodiversity.org/about-fes/

http://indianexpress.com/article/india/india-news-india/planting-a-
seed-of-hope/

http://www.business-standard.com/article/current-affairs/an-
informed-way-of-life-117042200028_1.html

8
WWW.INSIGHTSIAS.COM
1
INS INSIGHTS IAS PRELIMS TEST SERIES - 2017

91.Consider the following about the Centre for Development of


Telematics (C-DOT).
1. It was established to develop state-of-the-art
telecommunication technology to meet the needs of the Indian
telecommunication network.
2. It works in the niche of establishing air wave communication
facilities and expediting removal of inefficient fixed line
infrastructure by partnering with private companies.
Which of the above is/are correct?

a) 1 only
b) 2 only
c) Both 1 and 2
d) None
Solution: a)
Justification: Statement 1: It is the Telecom Technology development
centre of the Government of India. It was established in August 1984 as
an autonomous body.

Statement 2: Nearly 50% of present fixed line infrastructure, after


allowing MNCs entry into the Telecom Market is from C-DOT technology
and that in itself is a testimony to the Centre achieving its objectives
fully.

It builds partnerships and joint alliance with academia, industry,


solution providers, Telcos and other R&D organizations to offer cost
effective solutions.

Q Source: http://www.dot.gov.in/ : Autonomous Bodies

92. Consider the following statements.


8
WWW.INSIGHTSIAS.COM
2
INS INSIGHTS IAS PRELIMS TEST SERIES - 2017

1. If interest payments are subtracted from gross fiscal deficit, the


remainder will be effective revenue deficit.
2. If expenditure towards grants to states and UTs is subtracted
from gross fiscal deficit, the remainder will be gross primary
deficit.
Which of the above is/are correct?

a) 1 only
b) 2 only
c) Both 1 and 2
d) None
Solution: d)
Justification: Statement 1: It will be Gross Primary Deficit, which is
Gross Fiscal Deficit less interest payments. Net Primary Deficit is Net
Fiscal Deficit minus net interest payments. Net interest payment is
interest paid minus interest receipt.

A shrinking primary deficit indicates progress towards fiscal health.

Statement 2: It will be Effective Revenue deficit, which is a new term


introduced in the Union Budget 2011-12.

• While revenue deficit is the difference between revenue receipts


and revenue expenditure, the present accounting system includes
all grants from the Union Government to the state
governments/Union territories/other bodies as revenue
expenditure, even if they are used to create assets.
• According to the Finance Ministry, such revenue expenditures
contribute to the growth in the economy and therefore, should not
be treated as unproductive in nature
Q Source: 12th Macroeconomics NCERT: Public Finance

93. Consider the following statements.


1. M.S. Subbulakshmi founded the Indian Home Rule Society.
8
WWW.INSIGHTSIAS.COM
3
INS INSIGHTS IAS PRELIMS TEST SERIES - 2017

2. Chaitanya Mahaprabhu was an ardent opponent of the


Vaishnava School of Bhakti Yoga.
3. Jagjivan Ram founded the All India Depressed Classes League.
Select the correct answer using the codes below.

a) 3 only
b) 1 and 2 only
c) 1 and 3 only
d) 1, 2 and 3
Solution: a)
Justification: Statement 1: It was Shyamji Krishan who founded the
Indian Home Rule Society, India House and The Indian Sociologist in
London.

As an admirer of Lokmanya Tilak he supported him during the Age of


Consent bill controversy of 1890.

M.S. Subbulakshmi: She is Bharat Ratna Carnatic Musician, the United


Nations issued a stamp to mark the birth centenary of M.S.
Subbulakshmi recently.

Subbulakshmi performed at the UN fifty years ago, making her the first
Indian to have the honour.

Statement 2: He was a distinguished promoter for the Vaishnava School


of Bhakti Yoga.

• The Bhakti movement initiated by him strove to eradicate the evils


of caste and feudal systems of medieval India. He propagated the
worship of Shri Krishna and popularized the chanting of “Hare
Krishna Mantra” to all without any discrimination.
• Ministry of Culture has released a Commemorative Non-
Circulation Coin of ₹ 500 and a Circulation Coin of ₹ 10 as part of
year-long celebrations of commemoration of “500th Anniversary of
Shri Krishna Chaitanya Mahaprabhu’s Coming to Vrindavan”.
Statement 3: In 1934, he founded the Akhil Bhartiya Ravidas
Mahasabha in Calcutta and the All India Depressed Classes League.

8
WWW.INSIGHTSIAS.COM
4
INS INSIGHTS IAS PRELIMS TEST SERIES - 2017

• Through these Organizations he involved the depressed classes in


the freedom struggle.
• In 1935, Babuji appeared before the Hammond Commission at
Ranchi and demanded, for the first time, voting rights for the
Dalits.
Q Source: Insights Arts and Culture PT Exclusive Module 2017

94. The constitution of India empowers which of these


authorities to modify the list of notified scheduled castes as first
notified by the President of India?
a) Parliament
b) National Commission for SCs
c) Chairman, National Human Rights Commission (NHRC)
d) Cabinet Secretary
Solution: a)
Learning: The Parliament of India can modify the list of notified
scheduled castes, which upon receiving the assent of the President of
India can be notified as Scheduled Castes.

• Under the provision of Article 341, list of SCs in relation to a


states/UT is to be issued by a notified Order of the President after
consulting concerned state Government.
• Any subsequent inclusion in or exclusion from the list of Scheduled
Castes can be effected only through an Act of Parliament.
• Recently Sualgiri and Swalgiri communities of Odisha have been
notified as Scheduled Castes after President gave his assent to the
Constitution (Scheduled Castes) Order (Amendment) Act, 2017.
Q Source: http://www.hindustantimes.com/india-news/odisha-s-
sualgiri-swalgiri-communities-notified-as-scheduled-castes/story-
r5SNhzwv94zHtHIJfarJPL.html

8
WWW.INSIGHTSIAS.COM
5
INS INSIGHTS IAS PRELIMS TEST SERIES - 2017

95. Which of these mountain ranges is NOT among the ones


separating India from rest of Asia?
a) Hindukush Mountains
b) Karakoram Mountains
c) Himalayas
d) Arakanyoma
Solution: d)
Justification: The Indian sub-continent is a distinct geographic entity.

• It is separated from the rest of Asia in the north by a chain of very


high mountains, namely, the Hindukush. Sulaiman-Kirthar,
Karakoram and the Himalayas.
• All these mountain ranges radiate out from the Pamir Knot, which
lies to the north of the Hindukush Mountains.
• The Pamirs consist of a great mass of mountains with several deep
canyons and rolling plateaus.
• The North-east Himalayas separate it from the countries of South-
east Asia.

8
WWW.INSIGHTSIAS.COM
6
INS INSIGHTS IAS PRELIMS TEST SERIES - 2017

Q Source: Longman ICSE 10th Standard Geography: Chapter 2

96. Consider the following about Heart of Asia Conference.


1. It is being conducted since the first non-alignment conference at
Belgrade.
2. India has never hosted a Heart of Asia conference.
Which of the above is/are correct?

a) 1 only
b) 2 only
c) Both 1 and 2
d) None

8
WWW.INSIGHTSIAS.COM
7
INS INSIGHTS IAS PRELIMS TEST SERIES - 2017

Solution: d)
Justification: Statement 1: The Heart of Asia-Istanbul Process was
launched in 2011 and the participating countries include Pakistan,
Afghanistan, Azerbaijan, China, India, Iran, Kazakhstan, Kyrgyzstan,
Russia, Saudi Arabia, Tajikistan, Turkey, Turkmenistan and the United
Arab Emirates.

It keeps Afghanistan at its centre focus.

Statement 2: It provides a platform for discussing key regional issues


among participating States.

• The 6th Ministerial Conference (2016) of Heart of Asia summit was


held in Amritsar, India.
• The Amritsar Declaration recognises terrorism as the biggest
threat to peace and security.
• The declaration states the urgency to respond to the nexus between
drug menace and its financial support for terrorist entities in
Afghanistan.
Q Source: http://indianexpress.com/article/india/heart-of-asia-
declaration-full-text/

97. Cosmic rays have sufficiently high energy to disintegrate


every organic compound on which they fall. Why then their effect
on earth is not felt greatly?
1. They cannot penetrate matter surrounded by gases.
2. They are destroyed by sunlight in upper troposphere.
Which of the above is/are correct?

a) 1 only
b) 2 only
c) Both 1 and 2
d) None

8
WWW.INSIGHTSIAS.COM
8
INS INSIGHTS IAS PRELIMS TEST SERIES - 2017

Solution: d)
Justification: None of the above is correct.

Particles that bombard the Earth from anywhere beyond its atmosphere
are known as cosmic rays.

• But, fortunately, they are trapped in stratosphere and only a little


amount reaches the earth. This is why the damage on earth isn’t
visible.
• Cosmic rays constantly rain down on Earth, and while the high-
energy "primary" rays collide with atoms in the Earth's upper
atmosphere and rarely make it through to the ground, "secondary"
particles are ejected from this collision and do reach us on the
ground.
• According to NASA, cosmic rays therefore come equally from all
directions of the sky. So scientists are trying to trace back cosmic
ray origins by looking at what the cosmic rays are made of.
You can read more here https://helios.gsfc.nasa.gov/cosmic.html and
here http://www.space.com/32644-cosmic-rays.html

Q Source: As stated above

98. Consider the following about National Data Sharing and


Accessibility Policy (NDSAP).
1. It exempts government generated data from the requirement of
standardization.
2. It provides for open access to patents generated by private
companies which has substantial public interest.
3. It aims to facilitate availability of specific data in machine
readable form through a wide area network all over the country.
Select the correct answer using the codes below.

a) 1 and 2 only
b) 3 only

8
WWW.INSIGHTSIAS.COM
9
INS INSIGHTS IAS PRELIMS TEST SERIES - 2017

c) 1 and 3 only
d) 2 only
Solution: b)
Background: Large volumes and different types of data, including
some of scientific and technical relevance are generated and compiled by
various arms of the Government of India and various State Governments
for meeting their specific requirements.

Scientific organizations generate data and develop scientific data bases


deploying huge public funds. Since such data are not generated under
any standardized format, inter-operability of both scientific and
technical data poses a serious challenge.

Justification: Statement 1: It will apply to all data and information


created, generated, collected and archived using public funds provided
by Government of India directly or through authorized agencies by
various Ministries, bodies etc.

Statement 2: NDSAP aims to provide an enabling provision and


platform for proactive and open access to the data generated by various
Government of India entities.

The principles on which data sharing and accessibility need to be based


include: Openness, Flexibility, Transparency, Quality, Security and
Machine-readable.

Statement 3: The objective of this policy is to facilitate access to


Government of India owned shareable data (along with its usage
information) in machine readable form through a wide area network all
over the country in a periodically updatable manner.

The Department of Science and Technology is serving the nodal


functions of coordination and monitoring of policy.

Q Source: https://data.gov.in/faqs

9
WWW.INSIGHTSIAS.COM
0
INS INSIGHTS IAS PRELIMS TEST SERIES - 2017

99. Consider the following statements.


1. When the legislatures of two or more states pass resolutions
requesting the Parliament to enact laws on a matter in the State
List, then the Parliament can make laws for regulating that
matter.
2. The Parliament can make laws on any matter in the State List
for implementing obligations arising out of international
treaties, agreements or conventions.
Which of the above is/are correct?

a) 1 only
b) 2 only
c) Both 1 and 2
d) None
Solution: c)
Justification: Statement 1: A law so enacted applies only to those
states which have passed the resolutions. However, any other state may
adopt it afterwards by passing a resolution to that effect in its legislature.

Such a law can be amended or repealed only by the Parliament and not
by the legislatures of the concerned states.

Statement 2: Some examples of laws enacted under the above provision


are United Nations (Privileges and Immunities) Act, 1947; Geneva
Convention Act, 1960; Anti-Hijacking Act, 1982 and legislations relating
to environment and TRIPS.

Q Source: Chapter on Parliament: Indian Polity: M Laxmikanth

100. According to Birdlife International, designation of Important


Bird Areas (IBAs) is based on standardized criteria, namely
9
WWW.INSIGHTSIAS.COM
1
INS INSIGHTS IAS PRELIMS TEST SERIES - 2017

1. Hold all natural population and no migratory or congregatory


bird population
2. Hold significant numbers of one or more globally threatened
bird species
3. Hold a suite of restricted-range species or biome-restricted
species
Select the correct answer using the codes below.

a) 2 only
b) 1 only
c) 2 and 3 only
d) 1 and 3 only
Solution: c)
Justification: Birds are excellent indicators of ecosystem health.

Criteria are that it should (i) hold significant numbers of one or more
globally threatened bird species, (ii) be one of a set of sites that together
hold a suite of restricted-range species or biome-restricted species and
(iii) have exceptionally large numbers of migratory or congregatory
birds.

Learning: The IBA programme of Birdlife International aims to


identify, monitor and protect a global network of IBAs for conservation
of the world's birds and associated biodiversity.

The IBAs contain a range of habitats, such as wetlands, mudflats,


microhabitats in biodiversity hotspots, grasslands and scrublands etc.

Q Source: http://wiienvis.nic.in/Database/IBA_8463.aspx

101. These trees or flowers which are part of India’s rich


biodiversity find mention in Ancient literature. Match them
appropriately.
9
WWW.INSIGHTSIAS.COM
2
INS INSIGHTS IAS PRELIMS TEST SERIES - 2017

1. Punnag: A. Samba Puranam

2. Vat: B. Lalitaa Sahasram

3. Bandhook: C. Shiva in his form of Dakshinamurti

Select the best match using the codes below.

a) 1B, 2C, 3A
b) 1C, 2A, 3B
c) 1&2 – C, 3A
d) 1A, 2B, 3C
Solution: a)
Justification: Statement 1: This flower is sacred to Lord Vishnu,
forming his garland. A fragrant flower, it adorns the hair of Goddess
Lalitambika in the Lalitaa Sahasram (a text from Brahmanda Purana).

There are many references to the Punnaga flower in the lyrics of Karnatic
Music, as a flower for worship and as adornment of various Gods.

Statement 2: Vat or the banyan tree, India's National Tree, is a symbol


of spiritual knowledge. Lord Shiva in his form of Dakshinamurti the
universal Teacher sits under a Vata vrksha and illumines the minds of
sages seated at his feet.

• In the great Cosmic Deluge, Pralaya, nothing survives of the entire


creation except for the Lord in the form of an infant Krishna
floating on a banyan leaf (vata-patra shaayi), sucking his toe, a
familiar theme for Tanjore paintings.
Statement 3: Bandhook is a beautiful orange-red flower which blooms at
noon. In Sanskrit literature, it is often used to symbolize glowing red
color

It finds mention in Surya Ashtakam (Octet to Sun God), which is taken


from ancient Hindu text Samba Puranam.

Q Source: ENVIS Portal – Extensions: Mythology associated with


important flowers - http://www.flowersofindia.net/mythology.html

9
WWW.INSIGHTSIAS.COM
3
INS INSIGHTS IAS PRELIMS TEST SERIES - 2017

9
WWW.INSIGHTSIAS.COM
4

Вам также может понравиться